Test 2 practice questions

Ace your homework & exams now with Quizwiz!

A 1-year-old child will receive her scheduled MMR vaccination shortly. The nurse should teach the child's parents that she may develop what possible adverse effect related to the administration of this medication? A) Cough and fever B) Pallor and listlessness C) Serum sickness D) Nausea and vomiting

Ans: A Feedback: Adverse effects associated with MMR vaccine include fever and cough. Nausea and vomiting, pallor and listlessness, and serum sickness are not among the noted adverse effects of the MMR vaccine.

A pregnant woman is experiencing nausea and vomiting in her first trimester of pregnancy. Which herbal agent has traditionally been used as an antiemetic? A) Ginger B) Garlic C) Ginkgo biloba D) Green tea

Ans: A Feedback: Ginger has been used to relieve nausea and vomiting during pregnancy. Garlic, Ginkgo biloba, and green tea are not recommended to relieve nausea and vomiting in pregnancy.

A 70-year-old patient is seen in the family practice clinic. Which of the following vaccines should be administered to prevent shingles? A) Zoster vaccine B) Haemophilus influenzae Type b (Hib) vaccine C) Human papillomavirus (HPV) D) Pneumococcal polyvalent

Ans: A Feedback: Zoster vaccine is administered to adults 60 years and older to prevent herpes zoster (shingles). The Haemophilus influenzae Type b is not administered to prevent herpes zoster. HPV and pneumococcal vaccine do not address the risk factors for shingles.

A couple have been trying unsuccessfully for nearly a year to become pregnant and have now sought fertility counseling. The nurse should be aware of what potential etiological factors related to infertility? Select all that apply. A) Absence of sperm B) Endometriosis C) Vaginitis D) Blocked fallopian tubes E) Fibromyalgia

Ans: A, B, D Feedback: In women, the most common causes are ovulation disorders, blocked fallopian tubes, endometriosis, and advanced maternal age, which affects egg quality and quantity. In men, causes include absence of sperm, declining sperm counts, testicular abnormalities, and ejaculatory dysfunction. Vaginitis cannot cause infertility, and fibromyalgia is not noted as a common etiological factor.

Sitagliptin (Januvia) is prescribed for a patient who has been diagnosed with type 2 diabetes. What is the action of sitagliptin (Januvia)? A) It blocks the S phase of the cell cycle. B) It slows the rate of inactivation of the incretin hormones. C) It is a synthetically prepared monosodium salt. D) It inhibits hydrogen, potassium, and ATPase.

Ans: B Feedback: Sitagliptin (Januvia) minimizes the rate of inactivation of the incretin hormones to increase hormone levels and prolong their activity. Sitagliptin does not block the S phase of the cell cycle. Sitagliptin is not a synthetically prepared monosodium salt. Sitagliptin does not inhibit hydrogen, potassium, and ATPase.

The college health nurse is providing health education for freshmen. Which of the following pieces of information about immunizations is applicable to individuals of this age group? A) The oral polio should be updated. B) The yearly administration of flu vaccine is recommended. C) The tetanus toxoid must be within 2 years. D) The administration of hepatitis A vaccine is mandatory.

Ans: B Feedback: An annual flu vaccine is recommended for all adults. The administration of oral polio will not need to be updated in this population. The administration of tetanus toxoid should be every 10 years. HAV vaccination is not mandatory.

A young man has received a diagnosis of androgen deficiency and has been prescribed testosterone. At clinic follow-up appointments, the nurse should prioritize which of the following assessments? A) Bladder ultrasound and urine testing for glucose and ketones B) Weight and measurement of blood pressure C) Hearing assessment and abdominal girth measurement D) Deep tendon reflexes and random blood glucose testing

Ans: B Feedback: For patients who are being treated with androgens, the nurse should regularly assess the patient's blood pressure and body weight, as these may be affected by androgens. Urine testing and bladder ultrasound are generally unnecessary. It is prudent to monitor blood glucose levels, but assessment of DTRs is not required.

A patient is receiving oxytocin (Pitocin). Which of the following is a maternal adverse effect of Pitocin? A) Acute confusion B) Hypertension C) Edema D) Inverted T wave

Ans: B Feedback: Hypertension is a maternal adverse effect of Pitocin. Confusion, edema, and inverted T wave are not adverse effects of Pitocin.

An 8-year-old boy has been diagnosed with a sex hormone deficiency and has begun a course of treatment with testosterone. What change in the boy's health status would necessitate a stop to the course of treatment? A) Excessive growth in height B) Signs of puberty C) Recurrent urinary tract infections D) Increased blood pressure

Ans: B Feedback: If premature puberty occurs in a boy being treated with androgens (e.g., precocious sexual development, enlarged penis), it is necessary to stop the drug. Changes in blood pressure, increased growth, and urinary tract infections are unlikely causes for the cessation in treatment.

A woman in her last trimester of pregnancy is diagnosed with toxemia. The patient is admitted to labor and delivery and suffers a seizure. What medication will be administered parenterally? A) Magnesium oxide B) Magnesium sulfate C) Potassium chloride D) Calcium gluconate

Ans: B Feedback: Magnesium sulfate is given parenterally for convulsions related to pregnancy. Magnesium oxide is given orally for mild hypomagnesemia. Potassium chloride is not given for convulsions related to pregnancy. Calcium gluconate is not given for convulsion related to pregnancy.

A health care worker has received her annual influenza vaccination and has remained at the clinic after administration so that the nurse may observe for adverse reactions. The worker complains of pain at the site of IM injection. The nurse should recommend which of the following? A) ASA B) Acetaminophen C) Meperidine (Demerol) D) Heat application

Ans: B Feedback: Most vaccines can cause fever and soreness at the site of injection. Acetaminophen (e.g., Tylenol) can be taken two to three times daily for 24 to 48 hours if needed to decrease fever and discomfort. Aspirin and heat are not explicitly recommended; opioids are not necessary.

A patient has suffered from several infections in the last 6 months and unexplained impaired wound healing. What assessment should the nurse prioritize? A) Assess for pain. B) Assess for nutritional deficiencies. C) Assess genetic tendency for infection. D) Assess for edema and decreased hemoglobin.

Ans: B Feedback: Nutritional deficiencies may impair the function of essentially every body function. Signs and symptoms include unintended weight loss, increased susceptibility to infection, and impaired wound healing. The development of infection and impaired wound healing would require assessment for pain but would not provide evidence of cause. The nurse would not be able to assess genetic tendency. Edema and decreased hemoglobin would not explain the etiology of suffering infections and impaired wound healing.

A patient is diagnosed with infertility. What medication may the nurse administer for the treatment of infertility? A) Thyrotropin alfa (Thyrogen) B) Pegvisomant (Somavert) C) Menotropins (Pergonal) D) Somatotropin (Humatrope)

Ans: C Feedback: Menotropins (Pergonal) is administered in combination with HCG to induce ovulation. Thyrotropin alfa (Thyrogen) is a synthetic formulation of TSH used as a diagnostic adjunct for serum thyroglobulin. Pegvisomant (Somavert) is a growth hormone receptor antagonist used in the treatment of acromegaly in adults. Somatotropin (Humatrope) is administered to children for impaired growth.

A middle-aged woman with a family history of breast cancer has told the nurse that she has begun taking 250 mg of vitamin E each day in an effort to prevent cancer. How should the nurse best respond to this woman's statement? A) "Good for you. You're to be commended for being so proactive with your health." B) "If you do this, it's best to take several smaller doses during the day rather than one big dose." C) "Unfortunately, high doses of vitamins haven't been shown to protect against disease." D) "This is actually quite a dangerous practice, and might increase, rather than decrease, your cancer risk."

Ans: C Feedback: People should never self-prescribe megavitamins, large doses of vitamins in excess of the recommended dietary allowance (RDA). This practice does not, however, increase an individual's risk of cancer.

A patient is scheduled for a thyroidectomy to treat thyroid cancer. What is the medication of choice administered preoperatively? A) Sodium iodide 131I (Iodotope) B) Methimazole (Tapazole) C) Propylthiouracil (PTU) D) Propranolol (Inderal)

Ans: C Feedback: Propylthiouracil (PTU) is administered preoperatively for thyroidectomy. Sodium iodide131 (Iodotope) is a radioactive iodide that is used to destroy thyroid tissue. Methimazole (Tapazole) is similar to PTU but is not the drug of choice preoperatively. Propranolol (Inderal) is used to treat cardiovascular conditions, such as hypertension. It is not used in the preoperative phase of thyroidectomy.

A patient who is being cared for in the intensive care unit (ICU) is receiving desmopressin. In light of the black box warning that accompanies this drug, the ICU nurse should prioritize the assessment of which of the patient's laboratory values? A) Hematocrit B) Platelets C) Sodium D) Calcium

Ans: C Feedback: The FDA has issued a black box warning stating that patients taking desmopressin can develop hyponatremia, leading to seizures. Consequently, assessment of sodium levels takes precedence over hematocrit, calcium, and platelet levels.

A hospital patient is to receive 4 units of regular insulin prior to lunch. The nurse knows that the lunch trays are usually distributed at approximately 12:15. The nurse should plan to administer the patient's insulin at what time? A) 12:15 B) 12:10 C) 11:45 D) 11:15

Ans: C Feedback: With regular insulin before meals, it is very important that the medication be injected 30 to 45 minutes before meals so that the insulin is available when blood sugar increases after meals.

A patient has suffered from hypoglycemia twice in the past week. She states she eats one meal per day and snacks the rest of the day. What patient education will you provide for this patient? A) She should limit her alcohol with meals. B) She should increase her caloric intake. C) She should increase her protein intake during snacks. D) She should not eat at sporadic times.

Ans: D Feedback: A regular dietary intake associated with the administration of insulin or oral hypoglycemic will prevent episodes of hypoglycemia. The patient should limit her alcohol consumption, but alcohol consumption does not contribute to hypoglycemia. The patient should not necessarily increase her caloric or protein intake. The patient should coordinate her exercise with her dietary intake, but dietary intake is not the cause of her hypoglycemia.

A primiparous woman was vigilant in avoiding medications and herbs during her pregnancy and states that she is similarly committed to protecting her baby's health now that she is breast-feeding. What principle should guide the woman's use of medications while breast-feeding? A) Very few medications are explicitly contraindicated while breast-feeding. B) It is generally safer to use herbs rather than medications while breast-feeding. C) Most women can resume their prepregnancy medication regimen after delivery. D) Most medications are contraindicated while a woman is breast-feeding.

Ans: D Feedback: A wide variety of medications are contraindicated during pregnancy, and herbs are not guaranteed to be safe.

An 8-month-old infant is admitted to the pediatric floor of the community hospital with a new diagnosis of diabetes. The patient is to receive 1 unit of regular insulin subcutaneously. How will that 1 unit be administered? A) It is administered orally. B) It is administered as U-5. C) It is administered with a TB syringe. D) It is administered as U-10.

Ans: D Feedback: An infant should receive the dosage in a dilution strength of U-10. It is not administered orally, as U-5, or in a TB syringe.

Following the completion of diagnostic testing, an adult patient has received a diagnosis of hyperthyroidism. What nursing diagnosis should the nurse prioritize in this patient's care? A) Risk for hypothermia related to hyperthyroidism B) Constipation related to hyperthyroidism C) Risk for imbalanced nutrition: less than body requirements related to hyperthyroidism D) Anxiety related to hyperthyroidism

Ans: D Feedback: Anxiety is characteristic of a hyperthyroid state. Constipation, cold intolerance, and loss of appetite are associated with hypothyroidism.

When instructing the parents of a child who has received immunization in the vastus lateralis, which reaction is most common in the days after the administration? A) Nausea, vomiting, and diarrhea B) Rash and edema C) Weakness and difficulty walking D) Tenderness and redness at the site

Ans: D Feedback: Pain, tenderness, and redness at the injection site are associated with the administration of immunizations. Nausea, vomiting, diarrhea, rash, edema, weakness, or difficulty walking are not general reactions to immunizations.

A patient's medication administration record specifies that the patient is to receive 20 units of NPH insulin at 08:00. Before administering this medication, the nurse must do which of the following? A) Massage the chosen injection site. B) Assess the patient's understanding of diabetes. C) Assess the patient's urine for the presence of glucose. D) Have a colleague confirm the dosage.

Feedback: Before administering insulin, patient safety requires that two nurses always check the dosage. Assessing the patient's understanding of the disease may or may not be appropriate or necessary at this time. Injection sites are not massaged before administration. It is not necessary to assess urine for the presence of glucose.

A 72-year-old patient has been diagnosed with benign prostatic hypertrophy (BPH) and will begin treatment with tamsulosin (Flomax). The patient admits that he is skeptical about the drug's ability to relieve his distressing symptoms. The nurse should respond in what way? A) "Flomax can increase the amount of urine your kidneys produce, resulting in better urine flow." B) "Flomax can relax your prostate and your bladder neck, making it easier to pass urine." C) "Flomax makes your urine less alkaline, reducing the irritation that makes your prostate enlarge." D) "Flomax increases the strength of your bladder muscle and results in a stronger flow of urine."

Feedback: Tamsulosin works by relaxing the muscles in the prostate and bladder neck, enhancing the ability to urinate. Blockage of alpha1-adrenergic receptors decreases vascular smooth muscle contraction, influencing the activity of genitourinary smooth muscle. The drug does not change the pH of urine, strengthen the bladder muscle, or increase urine production.

A patient's current drug regimen includes intranasal administration of desmopressin acetate (DDAVP, Stimate). In this patient's plan of nursing care, what is the most likely desired outcome of treatment? A) The patient's urine output will not exceed 80 mL/h. B) The patient will gain 2 cm in height over the next 6 months. C) The patient will ovulate at least once over the next 8 weeks. D) The patient's vertical growth will cease in the next 4 weeks.

Ans: A Feedback: Desmopressin acetate (DDAVP, Stimate) is the prototype posterior pituitary hormone medication. It is used to treat diabetes insipidus. Consequently, reduced urine output is the primary goal of treatment. The drug does not influence growth or ovulation.

An older adult patient has been on oral corticosteroids to decrease the symptoms of chronic obstructive pulmonary disease. What is the patient at risk for developing? A) Osteoporosis B) Osteoarthritis C) Oat cell carcinoma D) Paget's disease

Ans: A Feedback: Drugs used for hypercalcemia, such as corticosteroids, place the patient at risk for developing osteoporosis due to the demineralization of the bone. The patient is not at risk for developing osteoarthritis or oat cell carcinoma. Paget's disease is an inflammatory condition and is not caused by corticosteroid use.

A 79-year-old woman has been diagnosed with osteoporosis, and the nurse is reviewing the risks and benefits of Fosamax, which her care provider has prescribed. The patient should be instructed to seek prompt medical attention if she develops which of the following? A) Esophageal bleeding B) Fever C) Sudden fatigue D) Musculoskeletal pain

Ans: A Feedback: Esophageal erosion and ulcers can result in hemorrhage, which is a medical emergency. Fever, fatigue, and musculoskeletal pain warrant follow-up, but none is an emergency.

A patient's fluid overload has required several doses of furosemide (Lasix). Which medication will be administered if the patient's serum potassium level is 2.8 mEq/L? A) Potassium chloride B) Calcium gluconate C) Potassium carbonate D) Sodium polystyrene sulfonate (Kayexalate)

Ans: A Feedback: Furosemide is a potassium-depleting diuretic in which potassium chloride will be administered to prevent hypokalemia. Calcium gluconate and zinc gluconate will not be administered to the patient. Sodium polystyrene sulfonate would be administered with hyperkalemia not hypokalemia.

A patient of Italian descent has been prescribed antidiabetic medications. Heavy intake of which of the following herbs should be avoided by this patient? A) Garlic B) Anise C) Basil D) Oregano

Ans: A Feedback: Garlic has been known to cause hypoglycemia when taken with antidiabetic medications. Anise, basil, and oregano are not noted to carry this risk.

A young man has begun weight training. Which hormone will be released in response to the increase in the size and number of muscle cells? A) Growth hormone B) Adrenocorticotropic hormone C) Thyroid-stimulating hormone D) Corticotropin-releasing hormone

Ans: A Feedback: Growth hormone stimulates the growth of body tissues, including growth of muscle cells. Corticotropin-releasing hormone causes the release of corticotropin (adrenocorticotropic hormone) in response to stress. Thyroid-stimulating hormone regulates the secretion of thyroid hormones.

Medroxyprogesterone acetate (Provera) is indicated for the treatment of women with A) uterine bleeding. B) cervical cancer. C) ovarian cancer. D) fibromyalgia.

Ans: A Feedback: Health care providers use progestins to suppress ovarian function in dysmenorrhea, endometriosis, endometrial cancer, and uterine bleeding. Progestins are not normally used to treat cervical cancer, ovarian cancer, and fibromyalgia.

A young woman is being seen in the gynecology clinic of the local health department. She has decided to begin hormonal contraceptives. What action is specific to hormonal contraceptives and should be taught to this woman? A) The cervical mucus is made resistant to penetration by spermatozoa. B) The follicle-stimulating hormone release is increased to block fertility. C) The maturation of the endometrial lining is activated by contraceptive. D) The pituitary gland increases the synthesis and release of luteinizing hormone.

Ans: A Feedback: Hormonal contraceptives inhibit the hypothalamic secretion of gonadotropin-releasing hormone, which inhibits FSH and LH. The drugs produce cervical mucus that resists penetration of spermatozoa into the upper reproductive tract. Hormonal contraceptives inhibit the release of follicle-stimulating hormone and luteinizing hormone. The drugs interfere with endometrial maturation and reception of ova that are released and fertilized.

A patient has been diagnosed with hyperplasia of the parathyroid gland. Which of the following alterations in blood values will you expect to observe? A) Hypercalcemia B) Hypoparathyroidism C) Hyperthyroidism D) Hypocalcemia

Ans: A Feedback: Hyperparathyroidism is most often caused by a tumor or hyperplasia of a parathyroid gland. Hypoparathyroidism, hyperthyroidism, and hypothyroidism are not associated with hyperplasia of the parathyroid gland. Clinical manifestations and treatment of hyperparathyroidism are the same as those of hypercalcemia.

A 5-year-old child is diagnosed with hypothyroidism. What will the patient likely develop if the condition is left untreated? A) Mental retardation B) Renal dysfunction C) An immune disorder D) Paralytic ileus

Ans: A Feedback: If cretinism is untreated until the child is several months old, permanent mental retardation is likely to result. The patient will not suffer from renal dysfunction, an immune disorder, or paralytic ileus.

A woman was administered misoprostol (Cytotec) in an effort to induce labor, but the care team is unsatisfied with the results. Consequently, oxytocin will be used. Prior to administering oxytocin, what must occur? A) Four hours must elapse after the last dose of misoprostol. B) The woman must have a type and cross-match performed. C) The woman must receive a bolus of 500-mL normal saline. D) The woman must have her electrolytes measured.

Ans: A Feedback: If the course of treatment changes and oxytocin is to be given after misoprostol, it is essential to wait 4 hours from the last administration of misoprostol before starting oxytocin. Blood work and IV hydration are not necessary.

A patient diagnosed with pernicious anemia asks why vitamin B12 is administered by injection. What is the best response to the patient? A) "Oral forms of vitamin B12 will not be absorbed." B) "Oral ingestion of vitamin B12 causes irritation and bleeding." C) "Pernicious anemia causes changes in the mucous membrane." D) "In severe pernicious anemia, vitamin B12 is not effective."

Ans: A Feedback: In pernicious anemia, vitamin B12 must be given by injection because oral forms are not absorbed from the GI tract. Vitamin B12 is not administered orally and thus will not cause irritation and bleeding. Pernicious anemia causes changes in the mucous membrane, but these changes do not explain why vitamin B12 is administered by injection.

A patient has an increased serum potassium level and has developed a cardiac arrhythmia. How do insulin and glucose assist in decreasing the serum potassium level? A) Insulin and glucose drive potassium into the cells. B) Insulin and glucose bind potassium to sodium. C) Insulin and glucose increase urinary excretion of potassium. D) Insulin and glucose increase fecal excretion of potassium.

Ans: A Feedback: Insulin and glucose lower serum potassium levels by driving potassium into the cells. Insulin and glucose do not bind potassium to sodium or increase urinary excretion of potassium or fecal excretion of potassium.

A 55-year-old female patient was successfully treated for thyroid storm earlier in the year. In subsequent health education, the nurse should caution the patient against excessive intake of what foods? A) Seafood and kelp B) Leafy green vegetables and beef C) Purine-rich foods D) Red wine and aged cheeses

Ans: A Feedback: Iodine is associated with thyroid storm and is present in foods (especially seafood and kelp) and in radiographic contrast dyes. Reports of iodine-induced hyperthyroidism have been reported after ingestion of dietary sources of iodine.

A middle-aged patient is proud of the fact that she is proactive with her health maintenance and tells the nurse that she has been taking 2400 International Units of vitamin D daily, stating, "More is better when it comes to vitamins." The nurse should provide health teaching to the patient because her current vitamin D intake creates a risk for A) hypercalcemia. B) hyperphosphatemia. C) hypocalcemia. D) tetany.

Ans: A Feedback: It is important to take vitamin D supplements cautiously and not overuse them; excessive amounts can cause serious problems, including hypercalcemia. Tetany is associated with hypocalcemia. Hypophosphatemia typically accompanies hypercalcemia.

When teaching young women about the hormones of reproduction, the nurse explains how the follicle matures and ruptures in ovulation. Which hormone is needed for this change? A) Luteinizing hormone (LH) B) Thyrotropin (TSH) C) Follicle-stimulating hormone (FSH) D) Leydig's cells

Ans: A Feedback: LH is important in the maturation and rupture of the ovarian follicle. TSH regulates the secretion of thyroid hormones. FSH stimulates the functions of sex glands. Leydig's cells are stimulated by LH in men to secrete androgens.

The results of a young man's plasma corticotropin (ACTH) stimulation test have confirmed a diagnosis of Addison's disease. The nurse has consequently provided health education around the necessity of hormone replacement therapy. The patient has asked the nurse, "How long am I going to have to take these medications?" How should the nurse best respond? A) "You'll need to take these for the rest of your life." B) "You'll have to take these until you can pass the corticotropin stimulation test without them." C) "You'll have to take these until your kidneys regain their normal level of function." D) "You'll need to take these until your symptoms have been stable for at least 6 months."

Ans: A Feedback: Lifetime hormone replacement is necessary in persons with Addison's disease.

A patient has undergone a thyroidectomy in which his parathyroid glands were also inadvertently removed. The patient's morning blood work reveals a serum calcium level of 3.1 mg/dL (normal 8.5 to 10.5 mg/dL). The nurse should consequently assess this patient's A) muscle tone. B) level of consciousness. C) respiratory rhythm. D) respiratory rate.

Ans: A Feedback: Low calcium levels can result in tetany. Decreased LOC and altered respiratory function are not characteristic of hypocalcemia.

A patient who regularly takes metformin has developed a severe infection. How will the infection change his diabetic regimen? A) Metformin is contraindicated in the presence of an infection. B) Metformin will be given more frequently to decrease blood sugar. C) Metformin will result in better regulation of blood sugar. D) Metformin will allow the patient to decrease the absorption of glucose.

Ans: A Feedback: Metformin is contraindicated in patients with diabetes complicated by fever, severe infections, severe trauma, major surgery, acidosis, and pregnancy. Metformin will not be given more frequently to decrease blood sugar. Metformin should be discontinued and is not administered for better regulation of blood sugar. Metformin will not decrease the absorption of glucose in this patient.

A primiparous woman tells the nurse that she and her partner are highly reluctant to have their infant vaccinated, stating, "We've read that vaccines can potentially cause a lot of harm, so we're not sure we want to take that risk." How should the nurse respond to this family's concerns? A) "Vaccinations are not without some risks, but these are far exceeded by the potential benefits." B) "The potential risks of vaccinations have been investigated and determined to be nonexistent." C) "Unfortunately, state laws mandate that your child receive the full schedule of vaccines." D) "Vaccines indeed cause several serious adverse effects, but these are usually treated at the site where your child receives the vaccination."

Ans: A Feedback: Mild reactions to vaccinations are common, but serious reactions are rare. Consequently, the benefits of vaccinations exceed the risks. Parental choice is still respected in most instances.

A patient suffers from hyponatremia, hypoglycemia, and lactic acidosis. What condition is most consistent with this clinical presentation? A) Myxedema coma B) Psychotic depression C) Cretinism D) Congenital hypothyroidism

Ans: A Feedback: Myxedema coma is severe, life-threatening hypothyroidism characterized by coma, hypothermia, cardiovascular collapse, hypoventilation, and severe metabolic disorders, such as hyponatremia, hypoglycemia, and lactic acidosis.

An older adult has been admitted to the hospital with failure to thrive. Assessment suggests that the patient is in a severely malnourished state. Which of the following assessment findings is most suggestive of a vitamin A deficiency? A) Night blindness B) Manic episodes C) Nonblanching erythema D) Petechiae

Ans: A Feedback: Night blindness is characteristic of vitamin A deficiency. Mania and integumentary changes are not normally associated with this vitamin deficiency.

A patient asks the nurse why a quick-acting sugar given by mouth is better in the regulation of insulin than the use of intravenous glucose for a low blood sugar. Which of the following statements by the nurse represents the most appropriate response to this question? A) "The ingestion of food allows the digestive tract to stimulate vagal activity and the release of incretins." B) "The combination of insulin and food will yield a higher blood sugar than intravenous glucose." C) "Both food and intravenous glucose will produce changes similarly in the gastrointestinal tract to increase blood sugar." D) "You are mistaken. The intravenous glucose yields a higher blood glucose through the release of incretins."

Ans: A Feedback: Oral glucose is more effective than intravenous glucose because glucose or food in the digestive tract stimulates vagal activity and induces the release of gastrointestinal hormones called incretins. The combination of insulin and food does not yield a higher blood glucose than intravenous insulin. Food stimulates the vagal nerve activity, but intravenous glucose does not. A statement that indicates that the patient is mistaken will belittle the patient and should be rephrased.

Laboratory testing reveals that a patient is in a severely hyperthyroid state and propylthiouracil (PTU) has been prescribed. When providing health education related to this drug, what should the nurse teach the patient? A) "You'll need to take these pills every 8 hours." B) "I'm going to show you how to perform your daily injection." C) "At first, you'll come to the outpatient clinic for your IV infusion once per day." D) "It's best to inject PTU into your abdomen rather than your arm."

Ans: A Feedback: PTU is well absorbed with oral administration. Because the half-life in the thyroid is relatively short, PTU must be given every 8 hours.

A patient with a diagnosis of diabetes is prescribed pramlintide (Symlin). How will this drug assist in controlling the patient's blood sugar? A) It slows gastric emptying. B) It blocks the absorption of food. C) It is absorbed by insulin. D) It increases the release of insulin.

Ans: A Feedback: Pramlintide slows gastric emptying, helping to regulate the postprandial rise in blood sugar. Pramlintide does not block the absorption of food. Pramlintide is not absorbed by insulin. Pramlintide does not increase the release of insulin.

A patient is to be administered an immunization. The serum contains aluminum phosphate. What route is most appropriate to administer this immunization? A) Intramuscularly B) Subcutaneously C) Intravenously D) Orally

Ans: A Feedback: Products containing aluminum should be given intramuscularly only because they cannot be given intravenously and greater tissue irritation occurs with subcutaneous injections. Immunizations containing aluminum are not administered orally.

A patient with a complex medical history is considering the use of oral contraceptives. The nurse should be aware that many antibiotics and antiseizure medications cause what effect when combined with oral contraceptives? A) Increased risk of pregnancy B) Increased blood pressure C) Increased risk of thromboembolism D) Increased gastric acid

Ans: A Feedback: Several medications may reduce the effectiveness of oral contraceptives (i.e., increase the likelihood of pregnancy). These include several antibiotics and antiseizure medications. This combination does not increase blood pressure, risk of thromboembolism, or gastric acid secretion.

Somatropin has been prescribed for a child who has been diagnosed with growth hormone deficiency. When providing health education to the child's parents, what teaching point should then nurse prioritize? A) The correct technique for daily injections of the drug B) The importance of regular blood pressure monitoring C) The rationale for not crushing the extended-release tablet D) The need to limit the child's activity level for the duration of treatment

Ans: A Feedback: Somatropin is administered by daily injections. Blood pressure monitoring and activity limitations are not necessary.

A 74-year-old woman states that many of her peers underwent hormone replacement therapy (HRT) in years past. The woman asks the nurse why her primary care provider has not yet proposed this treatment for her. What fact should underlie the nurse's response to the woman? A) The risks of stroke and breast cancer are unacceptably high in women taking HRT. B) HRT was found to cause mood disturbances in many of the women who used it long term. C) HRT was found to be a significant risk factor for bone fractures and osteoporosis. D) The risk of chronic obstructive pulmonary disease were found to be significantly higher in women taking HRT.

Ans: A Feedback: Studies have demonstrated no evidence for HRT in secondary prevention of heart disease and showed increased rates of CHD, thromboembolic stroke, venous thromboembolism, dementia, and breast cancer, which outweigh the benefits of decreased risk of fracture and colon cancer. Mood disturbances and COPD were not significant factors.

A toddler lives in a home that was built in the 1960s. The child has been diagnosed with lead poisoning after eating chips of flaking paint. Which medication will be administered to decrease lead levels? A) Succimer (Chemet) B) Folic acid C) Deferoxamine (Desferal) D) Deferasirox (Exjade)

Ans: A Feedback: Succimer (Chemet) chelates lead to form water-soluble complexes that are excreted in the urine. Succimer is used to treat lead poisoning in children. Folic acid is not used to treat lead poisoning. Deferoxamine is not used to treat lead poisoning. Deferasirox is not used to treat lead poisoning.

A patient with suspected adrenal insufficiency has been administered 1 mcg of cosyntropin (Cortrosyn) as an IV bolus. Serum testing 30 minutes later reveals a markedly elevated level of cortisol. What conclusion should the nurse infer from this finding? A) The patient has normal adrenocortical function. B) The patient has primary adrenal insufficiency. C) The patient has secondary adrenal insufficiency. D) The patient has Addison's disease.

Ans: A Feedback: The low-dose test involves the administration of 1 mcg of cosyntropin (Cortrosyn) as an IV bolus. In people with normal adrenocortical function, an increase in cortisol occurs in 20 minutes.

A woman has been prescribed Climara, a transdermal estradiol patch. Which of the following should she be instructed by the nurse regarding the administration? A) Avoid prolonged sun exposure at the patch site due to increased plasma concentrations. B) The application of heat at the patch site will decrease effectiveness and result in pregnancy. C) The medication, when exposed to sunlight, can increase the risk of breast cancer development. D) Exposure of the medication to occasional cold will increase effectiveness with application once a month.

Ans: A Feedback: The total amount of drug absorbed and the resulting plasma drug concentrations from transdermal estrogen can increase during exposure to heat, so patients should be advised to avoid prolonged sun exposure in the area of the patch. The application of heat at the patch site will increase effectiveness. The medication's exposure to sunlight will not increase the risk of breast cancer. The medication's exposure to cold will not change its effectiveness.

A woman who is in the first trimester of her pregnancy has told the nurse, "I've stopped taking my blood pressure pill because I know it could harm the baby. Instead, I've started taking natural and herbal remedies." What nursing diagnosis is suggested by the woman's statement? A) Deficient knowledge related to drug and herbal effects during pregnancy B) Health-seeking behaviors related to protection of fetal health C) Acute confusion related to the potential teratogenic effects of herbs D) Effective therapeutic regimen management related to use of herbs rather than drugs

Ans: A Feedback: This patient is evidently unaware of the fact that herbs pose risks to her fetus and that such risks are not limited to drugs. The nurse should address this knowledge gap. Acute confusion suggests a deficit in cognitive processes, not a lack of relevant and accurate information.

A patient has had surgery. After the surgical procedure, the patient starts to shiver and chill. What hormone will be released in response to the reaction to cold? A) Thyroid-stimulating hormone B) Luteinizing hormone C) Corticotropin D) Somatostatin

Ans: A Feedback: Thyroid-stimulating hormone is released in response to stress, such as exposure to cold. Luteinizing hormone is released in response to gonadotropin-releasing hormone. Corticotropin stimulates the adrenal cortex to produce corticosteroids. Somatostatin inhibits the release of growth hormone.

A patient has a decreased calcium level. Which of the following will participate in raising the patient's calcium level? A) Vitamin D B) Vitamin C C) Ferrous sulfate D) Vitamin B12

Ans: A Feedback: Vitamin D is a fat-soluble vitamin that plays an important role in calcium and bone metabolism. Vitamin C, ferrous sulfate, and vitamin B12 do not play a role in calcium and bone metabolism.

A patient with a history of atrial fibrillation has been brought to the emergency department (ED) by his wife after inadvertently taking 22 mg of warfarin rather than 2 mg, which is the patient's prescribed daily dose. The ED nurse should anticipate the administration of A) vitamin K. B) vitamin B12. C) vitamin A. D) vitamin B1.

Ans: A Feedback: Vitamin K is used to reverse the effects of warfarin (Coumadin).

A patient with a traumatic head injury and a diagnosis of diabetes insipidus is being treated with intravenous vasopressin (Pitressin). What change in the patient's status would prompt the immediate cessation of the drug infusion? A) Infiltration at the IV site B) A sudden decrease in urine output C) The appearance of blood in the patient's urine D) A rapid increase in blood glucose

Ans: A Feedback: When administering vasopressin intravenously, it is essential to use extreme caution due to the risk of extravasation of the medication, leading to tissue necrosis. Decreased urine output is a goal of treatment. Blood in the urine and changes in blood glucose levels are atypical events.

A patient has been admitted in acute hypercalcemia and has been determined to have a serum calcium level of 12.9 mg/dL. The emergency department nurse's priority intervention is A) administration of IV calcitonin. B) administration of IV normal saline. C) oxygen supplementation. D) subcutaneous administration of exogenous parathyroid hormone (PTH).

Ans: B Feedback: Acute hypercalcemia is a medical emergency, and rehydration is a priority; it is essential to administer an IV saline solution. Calcitonin is not administered by the IV route. PTH is not administered exogenously and would exacerbate hypercalcemia.

A home care nurse observes that a client's supply of Synthroid has run out in less than half the time that it should have. What assessment findings would be most consistent with overuse of Synthroid? A) Constipation and abdominal distention B) Hyperactivity and insomnia C) Bradycardia and hypotension D) Joint pain and decreased mobility

Ans: B Feedback: Adverse effects of levothyroxine include signs and symptoms of hyperthyroidism. Other more serious adverse effects are tachycardia, cardiac dysrhythmias, angina pectoris, myocardial infarction, and heart failure. Nervousness, hyperactivity, insomnia, diarrhea, abdominal cramps, nausea, vomiting, weight loss, fever, and an intolerance to heat have also been reported. Bradycardia and constipation are associated with hypothyroidism.

An elderly patient's compromised nutritional status has necessitated the use of a nutritional formula. When reviewing this patient's laboratory findings, the nurse should prioritize which of the following values? A) Erythrocyte sedimentation rate (ESR) B) Serum albumin C) GGT, AST, and ALT D) Blood urea nitrogen

Ans: B Feedback: For patients receiving nutritional formulas, the nurse observes for weight gain and increased serum albumin. In a patient with compromised nutrition, there is less emphasis placed on BUN, liver enzymes, and ESR.

A patient with a diagnosis of Addison's disease has had fludrocortisone acetate (Florinef Acetate) added to his medication regimen by his endocrinologist. What is the most plausible rationale for this change in treatment? A) The patient's anterior pituitary gland is overstimulated. B) The patient requires increased mineralocorticoid supplementation. C) The patient's adrenal cortex has become resistant to hydrocortisone. D) The patient's GI tract is unable to tolerate hydrocortisone.

Ans: B Feedback: If a patient with Addison's disease requires additional mineralocorticoid supplementation, then fludrocortisone acetate (Florinef Acetate) may be indicated. Pituitary dysfunction, GI intolerance, and resistance to hydrocortisone are not plausible rationales.

An infant is being administered an immunization. Which of the following provides an accurate description of an immunization? A) It should be administered to a pregnant woman prior to the infant's birth. B) It is the administration of an antigen for an antibody response. C) It produces many adverse reactions, particularly autism, in the infant. D) It protects the infant from exposure to infectious antibodies.

Ans: B Feedback: Immunization involves administration of an antigen to induce antibody formation. Live, attenuated vaccines should not be administered to the pregnant woman. There are limited adverse effects associated with immunization. The immunization protects the infant from infectious antigens, producing an antibody response. It does not protect from antibodies.

A patient who has been taking oral prednisone for several months abruptly ceased treatment several days ago on the advice of a family member. The patient has now been admitted to the emergency department with signs and symptoms that are characteristic of addisonian crisis. When assessing this patient, the nurse should prioritize what assessment? A) Assessment of urine for ketones B) Assessment of serum potassium levels C) Assessment for adventitious lung sounds D) Assessment for venous thromboembolism

Ans: B Feedback: In addisonian crisis, the loss of sodium leads to retention of potassium, resulting in symptoms of hyperkalemia. As a result, vigilant monitoring of serum potassium levels is more important than respiratory effects, urinalysis, or hypercoagulation.

Ans: A Feedback: Because glyburide stimulates pancreatic beta cells to produce more insulin, it is effective only when functioning pancreatic beta cells are present. The presence of normal blood glucose levels would render the medication unnecessary. Self-administration is common but not absolutely necessary.

Ans: B Feedback: It is essential to discontinue metformin if renal impairment occurs. The other listed changes in laboratory values do not necessarily indicate that metformin should be discontinued.

A 20-year-old college student has presented to the campus medical clinic seeking to begin oral contraceptive therapy. The nurse has recognized the need for adequate health education related to the patient's request. The nurse should emphasize the fact that successful prevention of pregnancy depends primarily on the patient's A) current health status. B) vigilant adherence to the drug regimen. C) knowledge of sexual health. D) risk factors for adverse effects.

Ans: B Feedback: It is necessary to assess each patient's need and desire for contraception, as well as her willingness to comply with the prescribed regimen. Compliance is essential for effective contraception. Health status, knowledge, and risk factors are all valid focuses for health education, but these do not primarily determine success of therapy.

A 22-year-old woman began using oral contraceptives several months ago and has presented for an appointment to discuss recent worrisome changes in her health status. Which of the following changes in the woman's health may the nurse potentially attribute to the use of oral contraceptives? A) Fatigue B) Frequent high blood pressure readings C) Frequent headaches without aura D) Nausea and vomiting

Ans: B Feedback: It is very important to monitor for adverse drug effects such as high blood pressure, gallbladder disease, and blood-clotting disorders, which may be associated with oral contraceptive use. Headaches, nausea, and fatigue are not among the more common adverse effects of oral contraceptives.

A 3-year-old child has been diagnosed with an acute gastrointestinal infection that has caused her severe diarrhea for the past 36 hours. The pediatric nurse should anticipate the use of A) magnesium gluconate. B) Pedialyte. C) sodium bicarbonate. D) 50% dextrose.

Ans: B Feedback: Oral electrolyte solutions (e.g., Pedialyte) contain several electrolytes and a small amount of dextrose. They are especially useful in children for treatment of diarrhea and may prevent severe fluid and electrolyte depletion. Magnesium gluconate, 50% dextrose, and sodium bicarbonate are not typically used for this purpose.

The nurse is providing health education to an 80-year-old woman who has just been diagnosed with osteoporosis. Which of the following represents an accurate statement when teaching her about this diagnosis? A) "Osteoporosis is usually a result of a bone injury." B) "Osteoporosis causes a risk for fractures." C) "Osteoporosis results from nonmodifiable risk factors." D) "Osteoporosis occurs only in women."

Ans: B Feedback: Osteoporosis results when bone strength is impaired, leading to increased porousness and vulnerability to fracture. Osteoporosis is not a result of fracture. Osteoporosis is partially preventable. Osteoporosis is not limited to the female gender.

A woman in labor is being treated with magnesium sulfate intravenously and is beginning to show signs and symptoms of hypermagnesemia. The infusion has been discontinued, and the nurse should anticipate administration of what drug? A) Metoprolol (Lopressor) B) Calcium gluconate C) Potassium chloride D) Furosemide (Lasix)

Ans: B Feedback: Overdoses of magnesium sulfate may lead to hypotension, muscle paralysis, respiratory depression, and cardiac arrest. Calcium gluconate, the antidote for magnesium sulfate, should be readily available for use if hypermagnesemia occurs.

When administering magnesium sulfate, for what should the nurse assess the patient? A) Dry, pale skin B) Respiratory depression C) Agitation D) Tachycardia

Ans: B Feedback: Overdoses of magnesium sulfate may lead to hypotension, muscle paralysis, respiratory depression, and cardiac arrest. The nurse would not anticipate dry, pale skin; agitation; or tachycardia.

A 58-year-old patient who has been living with diabetes since age 14 states he has pain in his feet and hands. What is this pain most likely a result of? A) A diabetes-related infectious process B) Peripheral neuropathy C) An autoimmune disorder D) Hypertension resulting from diabetes

Ans: B Feedback: Pain in the feet and hands is related to changes in small blood vessels resulting in neuropathy. The long-term effect of diabetes can result in an infectious process, but the pain described is not indicative of an infection. Latent autoimmune diabetes of the adult has an onset in adulthood and thus is not a long-term disorder. Hypertension is a long-term chronic effect of diabetes but is not what has been described with pain in the feet and hands.

A patient with a history of homelessness has been diagnosed with nutritional deficits that include a vitamin E deficiency. What aspect of the patient's current health status would contraindicate the safe and effective use of vitamin E supplements? A) Cirrhosis B) Thrombocytopenia C) Low serum albumin D) IV opioid abuse

Ans: B Feedback: Patients with a history of bleeding disorders or thrombocytopenia should not take vitamin E. Opioid use and liver disease do not necessarily contraindicate the use of vitamin E supplements.

A patient has been diagnosed with acromegaly. Which of the following medications may be ordered to treat this endocrine disorder? A) Gonadorelin hydrochloride (Factrel) B) Octreotide (Sandostatin) C) Nafarelin (Synarel) D) Gonadorelin acetate (Lutrepulse)

Ans: B Feedback: Prescribers order octreotide for patients with acromegaly to reduce levels of GH. Gonadorelin hydrochloride, nafarelin, and gonadorelin acetate are not administered to treat acromegaly.

A patient suffers from dysmenorrhea. Which oral medication will be prescribed that has the ability to provide physiological actions on the neuroendocrine control of ovarian function? A) Estrogen B) Progestins C) Naprosyn (Naproxen) D) Ibuprofen (Motrin)

Ans: B Feedback: Progestins are used to suppress ovarian function in dysmenorrhea, endometriosis, endometrial cancer, and uterine bleeding. Estrogen is not administered alone, but in combination with progestins. Nonsteroidal anti-inflammatory agents, such as naproxen and ibuprofen, decrease pain and inflammation but do not affect ovarian function.

A patient is scheduled to begin a drug regimen for the treatment of hyperthyroidism. Prior to administering propylthiouracil (PTU), the nurse has reviewed the relevant black box warning and should teach the patient about the need for what form of follow-up? A) Serial complete blood counts B) Routine liver function testing C) Daily nonfasting blood glucose testing D) Chest radiographs every 3 months

Ans: B Feedback: The FDA has issued a black box warning for PTU stating that severe liver injury resulting in death or acute liver failure may occur within 6 months of treatment. All patients should receive instructions about the signs and symptoms of acute liver failure. Routine liver function testing to assess for liver failure is important. There is no specific need for chest radiographs, CBCs, or glucose monitoring.

A 26-year-old woman's gradual development of a "moon face," coupled with protracted weight gain, has caused the nurse practitioner to suspect the possibility of Cushing's disease. When explaining this health problem to the patient, how should the nurse describe it? A) "Cushing's disease happens when your pituitary gland doesn't stimulate your other glands enough." B) "Cushing's disease is a result of an overproduction of steroid hormones by your kidneys." C) "Cushing's disease most often happens when people are taking corticosteroid medications and stop them abruptly." D) "Cushing's disease often results from a growth on your kidney that cause inadequate production of steroids."

Ans: B Feedback: The cause of Cushing's disease is adrenocortical excess. Cessation of exogenous steroids can precipitate addisonian crisis. Dysfunction of pituitary gland is an atypical cause. Tumors may cause overproduction, not underproduction, of corticosteroids.

A nursing student is scheduled to receive the hepatitis B series. What type of immunity will this immunization provide? A) Active immunity B) Passive immunity C) Innate immunity D) Natural immunity

Ans: B Feedback: The hepatitis B series produces passive immunity. Passive immunity occurs when antibodies are formed by the immune system of another person or animal and transferred to the host. Active immunity is produced by the person's own immune system in response to a disease caused by a specific antigen or administration of an antigen from a source outside the body, usually by injection. Innate or natural immunity, which is not produced by the immune system, includes the general protective mechanisms.

A patient is seen in the emergency room for a laceration sustained on broken glass. The nurse assesses the patient for the last tetanus toxoid he received. How often should the patient be administered a tetanus toxoid? A) Every year B) Every 10 years C) Every 2 years D) Every 5 years

Ans: B Feedback: The patient should receive a tetanus toxoid every 10 years according to the CDC. It is not necessary to receive tetanus toxoid yearly, every 2 years, or every 5 years.

An 80-year-old woman has experienced recent declines in bone density and has consequently been deemed a candidate for treatment with alendronate (Fosamax). During health education, what teaching point should the nurse emphasize? A) "Your Fosamax will be most effective if you couple it with an increase in your consumption of dairy products." B) "It's important that you not lie down for half an hour after taking your Fosamax." C) "Make sure to let your care provider know promptly if you experience bone pain." D) "You'll have less stomach upset if you take your Fosamax with some bland food."

Ans: B Feedback: The person must remain upright (with head elevated 90 degrees if in bed, sitting upright in a chair, or standing) for at least 30 minutes after administration of Fosamax. It is always necessary to take the drug with a full glass of water, not juice or coffee, at least 30 minutes before breakfast and before taking other drugs. Bone pain is a possible adverse effect, but prevention of esophageal ulceration is a priority. Increased dairy consumption is not vital.

A woman is started on propylthiouracil (PTU). What is the primary mode of action for propylthiouracil? A) Destroys part of the thyroid gland B) Inhibits production of thyroid hormone C) Suppresses the anterior pituitary hormones D) Stimulates the thyroid cells

Ans: B Feedback: The thioamide drugs inhibit synthesis of thyroid hormone. Iodine preparations inhibit the release of thyroid hormones and cause them to be stored within the thyroid gland. Propylthiouracil does not destroy part of the thyroid gland. Propylthiouracil does not suppress the anterior pituitary hormones. Propylthiouracil does not sedate the central nervous system or suppress the cardiac output.

A patient is diagnosed with type 1 diabetes. What distinguishing characteristic is associated with type 1 diabetes? A) Blood glucose levels can be controlled by diet. B) Exogenous insulin is required for life. C) Oral agents can control blood sugar. D) The disease always starts in childhood.

Ans: B Feedback: Type 1 diabetes will result in eventual destruction of beta cells, and no insulin is produced. The blood glucose level can only be controlled by diet in type 2 diabetes. In type 2 diabetes, oral agents can be administered. Type 1 diabetes is diagnosed at many ages, not only in childhood.

A patient is being treated for hypothyroidism and has developed symptoms of adrenal insufficiency. What medication will be added to the patient's medication regimen? A) Anti-infective agent B) Corticosteroid agent C) Nonsteroidal anti-inflammatory agent D) Antiadrenergic agent

Ans: B Feedback: When hypothyroidism and adrenal insufficiency coexist, the adrenal insufficiency should be treated with a corticosteroid drug before starting thyroid replacement. Anti-infective agents, nonsteroidal anti-inflammatory agents, or antiadrenergic agents are not administered for adrenal insufficiency.

A woman who takes highly active antiretroviral therapy (HAART) for HIV/AIDS has become pregnant. What effect will the woman's pregnancy have on her drug regimen? A) She must discontinue HAART due to the risk of teratogenic effects. B) Her dosages of HAART must be increased to reduce the risk of in utero transmission. C) She can continue her HAART unchanged. D) Some components of her HAART must be replaced or discontinued.

Ans: C Feedback: Antiretroviral drug therapy for the pregnant woman reduces perinatal transmission by about two thirds. In general, highly active antiretroviral therapy, or HAART, is safe, with recommended dosage the same as for nonpregnant women.

An elderly adult woman has been diagnosed with postmenopausal osteoporosis and has been prescribed calcitonin. The nurse should anticipate administration by what route? A) Oral B) Topical C) Intranasal D) Intravenous

Ans: C Feedback: Calcitonin can be administered by the intranasal route for the treatment of osteoporosis. It is not given orally, topically, or intravenously.

After experiencing several months of worsening nocturia, a patient has been assessed for benign prostatic hypertrophy (BPH) and has begun drug treatment. In addition to nocturia, what other sign or symptom is most likely to accompany BPH? A) Hematuria B) Erectile dysfunction C) Urinary frequency D) Flank pain

Ans: C Feedback: Clinical manifestations of BPH include urinary frequency, hesitancy, urgency, dribbling, and decreased force of the urinary stream. Hematuria, ED, and flank pain are not characteristic of this problem.

A postmenopausal woman is administered estradiol (Estraderm). What condition will be prevented in this patient? A) Endometriosis B) Amenorrhea C) Osteoporosis D) Uterine cancer

Ans: C Feedback: Conjugated estrogens are used to prevent osteoporosis in postmenopausal women. Conjugated estrogens are not administered in postmenopausal women to treat endometriosis, amenorrhea, or uterine cancer.

A patient with iron deficiency anemia has begun taking daily supplements of oral ferrous sulfate. The nurse who is planning this patient's care should add what nursing diagnosis to the nursing care plan? A) Risk for excess fluid volume related to use of iron supplements B) Risk for unstable blood glucose related to use of iron supplements C) Risk for constipation related to use of iron supplements D) Risk for peripheral neurovascular dysfunction related to use of iron supplements

Ans: C Feedback: Constipation is a common adverse effect of iron supplements. Neurovascular dysfunction, fluid volume excess, and unstable blood glucose are unlikely to result from iron supplements.

An older adult resident of a long-term care facility has been prescribed calcium citrate to address her decreasing bone density. The nurse should review the resident's medication administration record knowing that what medication may decrease the effects of calcium? A) Hydrochlorothiazide B) Ibuprofen C) Prednisone D) Diltiazem (Cardizem)

Ans: C Feedback: Corticosteroids reduce the effects of calcium by various mechanisms. Thiazide diuretics have the opposite effect. NSAIDs and calcium channel blockers do not appreciably affect the pharmacokinetics of calcium supplements.

A patient is started on levothyroxine (Synthroid) for treatment of hypothyroidism. During patient teaching, how frequently can the dosage be increased until symptoms are relieved? A) Every 3 to 5 days B) Weekly C) Every 2 weeks D) Monthly

Ans: C Feedback: Dosage is influenced by the choice of drug. As a general rule, initial dosage is relatively small. Dosage is gradually increased at approximately 2-week intervals until symptoms are relieved. The dosage is not increased in 3 to 5 days. The dosage is not increased in 1 week. The dosage is not increased monthly.

A nurse is performing health education with a woman who has just learned that she is pregnant. The nurse has explained the concept of teratogenic drugs and emphasized the need to have her care provider assess any medications she should consider taking. The nurse should teach the woman that drug-induced teratogenicity is most likely to occur at what point in her pregnancy? A) During the second half of her third trimester B) In the 7 to 10 days after conception C) In the first trimester during organogenesis D) During 30 to 34 weeks of gestation

Ans: C Feedback: Drug-induced teratogenicity is most likely to occur when drugs are taken during the first three months of pregnancy, during organogenesis.

A woman suffers from amenorrhea. Which of the following medications will most likely be prescribed? A) Testosterone B) Follicle-stimulating hormone C) Estrogen D) Lactate

Ans: C Feedback: Estrogen is prescribed in woman of reproductive age to treat amenorrhea. Testosterone and FSH are not used for amenorrhea. Lactate is not used to treat amenorrhea.

A 37-year-old woman has a 10 pack-year smoking history and has been using oral contraceptives for the past 2 years. The nurse should plan health education interventions in the knowledge that this patient faces an increased risk of A) osteoporosis. B) dementia. C) myocardial infarction. D) deep vein thrombosis.

Ans: C Feedback: For women 35 years of age and older who smoke, there is an increased risk of myocardial infarction and other cardiovascular disorders, even with low-dose oral contraceptives. The increase in this risk supersedes that of osteoporosis, dementia, or DVT.

A hospital patient with a diagnosis of type 1 diabetes is ordered Humulin R on a sliding scale. Based on the patient's blood glucose reading, the nurse administered 8 units of insulin at 07:45. The nurse recognizes the need to follow up this intervention and will reassess the patient's blood glucose level when the insulin reaches peak efficacy. The nurse should consequently check the patient's blood glucose level at what time? A) 08:15 B) Between 08:45 and 09:45 C) Between 09:45 and 10:45 D) Between 11:15 and 11:45

Ans: C Feedback: Humulin R peaks between 2 and 3 hours after administration.

A child has received a diagnosis of Addison's disease, and the nurse is providing health education to the child's family around hormone replacement therapy. When planning this child's care, the nurse should know that hormone replacement therapy constitutes a risk for what nursing diagnosis? A) Latex allergic response B) Deficient fluid volume C) Delayed growth and development D) Impaired gas exchange

Ans: C Feedback: Hydrocortisone may affect growth velocity. It is unlikely to affect allergy response, or respiration. Fluid volume excess, not deficit, is a possibility.

An adult is diagnosed with an increased low-density lipoprotein cholesterol level. What hormone level may be low based on this diagnosis? A) Adrenocorticotropic hormone (ACTH) B) Oxytocin C) Growth hormone D) Somatostatin

Ans: C Feedback: In adults, deficient growth hormone can cause increased fat, reduced skeletal and heart muscle mass, reduced strength, reduced ability to exercise, and worsened cholesterol levels (increased low-density lipoprotein cholesterol). Adrenocorticotropic hormone, oxytocin, and somatostatin do not affect low-density lipoprotein.

A 55-year-old woman is diagnosed with a vitamin D deficit. What disorder results from this deficit? A) Chondromalacia B) Chondritis C) Osteomalacia D) Osteopenia

Ans: C Feedback: In adults, vitamin D deficiency causes osteomalacia, a condition characterized by decreased bone density and strength. Chondromalacia is characterized as a softening of the patella. Chondritis is an inflammation of the cartilage. Osteopenia is a decrease in bone tissue.

The nurse is closely following a patient who began treatment with testosterone several months earlier. When assessing the patient for potential adverse effects of treatment, the nurse should prioritize which of the following assessments? A) Skin inspection for growing lesions B) Lung function testing C) Assessment of serum calcium levels D) Assessment of arterial blood gases

Ans: C Feedback: In the patient taking testosterone, the nurse should monitor serum calcium levels and observe for signs of hypercalcemia (e.g., kidney stones, polyuria, abdominal pain, nausea, vomiting, depression). ABGs, respiratory function, and skin integrity are not typically affected by testosterone.

A patient with hypothyroidism is started on levothyroxine (Synthroid). What should the patient be taught regarding medication administration in the home setting? A) Take medication with milk or food. B) Do not exercise with the medication. C) Take the medication on an empty stomach. D) Levothyroxine has a short half-life.

Ans: C Feedback: Levothyroxine (Synthroid) should be taken on an empty stomach to increase absorption. The medication should not be taken with food. Exercise has no effect on the administration of the medication. Levothyroxine has a long half-life of about 6 to 7 days.

A nurse reviews the current medication administration record of a patient who has recently been prescribed octreotide. The nurse performs this important safety action in order to prevent drug interactions that could result in A) increased growth. B) anaphylaxis. C) cardiac complications. D) respiratory arrest.

Ans: C Feedback: Numerous medications combine with octreotide to result in a prolonged QT interval with an increased risk of ventricular dysrhythmia and cardiac arrest. Increased growth, anaphylaxis, and respiratory arrest are not noted adverse effects or the result of drug interactions with octreotide.

A woman has been unable to conceive for many months and will soon begin treatment with clomiphene (Clomid). What health education should the nurse provide to this patient? A) Avoid drinking alcohol while taking Clomid. B) Perform daily OTC pregnancy tests beginning the day after taking Clomid. C) Take her basal temperature between 5 to 10 days after taking Clomid. D) Report any numbness or tingling in her hands or lips to her care provider.

Ans: C Feedback: Ovulation occurs 5 to 10 days after the course of clomiphene treatment has been completed. Prior to beginning the drug regimen, the nurse instructs the woman about taking her basal temperature 5 to 10 days following administration. An incremental rise in temperature is an indication of ovulation. There is no specific contraindication against alcohol, and neurological adverse effects are not expected.

A patient is scheduled to receive an immunization. In which of the following patients may the administration of a live vaccine be contraindicated? A) Patient with renal insufficiency B) Patient with hepatic failure C) Patient taking steroid therapy D) Patient over the age of 65 years

Ans: C Feedback: Patients receiving a systemic corticosteroid in high doses (e.g., prednisone 20 mg or equivalent daily) or for longer than 2 weeks should wait at least 3 months before receiving a live-virus vaccine. No evidence supports withholding immunizations related to renal insufficiency or hepatic failure. Patients over the age of 65 should receive immunizations as needed to protect from infectious disease.

On the advice of his brother, a 53-year-old man has made an appointment to request a prescription for Viagra. The nurse who works at the clinic is reviewing the man's medical history and would recognize what health problem as being prohibitive to this treatment? A) Type 2 diabetes, treated with metformin B) Hypercholesterolemia, treated with simvastatin C) Angina, treated with nitroglycerin D) Hypertension, treated with metoprolol

Ans: C Feedback: Prescribers should not order Viagra for men who also take organic nitrates, commonly used to treat angina, because the sildenafil-nitrate combination can cause severe hypotension resulting in dizziness, syncope, heart attack, or stroke. Metformin, metoprolol, and simvastatin are not likely to cause adverse effects.

A nurse who provides care at a long-term care facility is preparing to administer a resident's scheduled dose of levothyroxine (Synthroid). What assessment should the nurse perform prior to administration? A) Level of consciousness and orientation B) Oxygen saturation level C) Heart rate D) Respiratory rate

Ans: C Feedback: Regular monitoring of blood pressure and pulse is essential in older adults receiving Synthroid. As a general rule, levothyroxine should not be given if the resting heart rate is more than 100 beats per minute.

A patient is to receive a medication that the nurse recognizes as an expectorant that is normally administered for the treatment of hyperthyroidism. Which medication will be administered? A) Propylthiouracil (PTU) B) Methimazole (Tapazole) C) Saturated solution of potassium iodide (SSKI) D) Sodium iodide131I (Iodotope)

Ans: C Feedback: Saturated solution of potassium iodide (SSKI) is more often used as an expectorant but may be given as a preparation for thyroidectomy. Propylthiouracil (PTU) is not administered as an expectorant. Methimazole (Tapazole) is not administered as an expectorant. Sodium iodide 131I (Iodotope) is not administered as an expectorant.

A patient is admitted to the hospital with hyperkalemia. The patient is prescribed sodium polystyrene sulfonate (Kayexalate). How does this medication lower the patient's potassium level? A) It decreases the absorption of exogenous potassium. B) It increases urinary excretion of potassium. C) It combines with potassium ions for elimination. D) It releases sodium to acidify urine with potassium.

Ans: C Feedback: Sodium polystyrene sulfonate (Kayexalate), a cation exchange resin, administered orally, removes potassium from the body in the stool. Kayexalate does not decrease the absorption of potassium. Kayexalate does not increase urinary excretion of potassium. Kayexalate does not release sodium to acidify urine with potassium.

A 13-year-old boy who is in the first percentile of height for his age has been referred for care. Which of the following assessment findings would contraindicate the safe and effective use of somatropin to treat his growth deficiency? A) The boy has not yet begun puberty. B) The boy has low serum albumin levels. C) The epiphyses of the boy's long bones have closed. D) The sutures of the boy's skull have not yet fully closed.

Ans: C Feedback: Somatropin is ineffective when impaired growth is present after puberty, when the epiphyses of the long bones have closed. Low albumin is not a contraindication to the use of somatropin. The sutures of the skull close early in life.

A patient is to be administered glipizide (Glucotrol). Which of the following factors would contraindicate the administration of glipizide (Glucotrol) to this patient? A) A diagnosis of hypertension B) The ingestion of carbohydrates C) Allergy to sulfonamides D) Increase in alkaline phosphatase

Ans: C Feedback: Sulfonylureas are contraindicated in patients with hypersensitivity to them, with severe renal or hepatic impairment, and who are pregnant. A diagnosis of hypertension does not cause contraindication of sulfonylureas. The patient should consume carbohydrates in association with the oral hypoglycemic agent. An increase in alkaline phosphatase does not result in the contraindication of glipizide (Glucotrol).

A patient with Cushing's disease will soon begin treatment with ketoconazole. When planning the patient's care, the nurse should be cognizant of the black box warning for this drug and consequently monitor what laboratory values? A) White blood cell differential B) Blood urea nitrogen and creatinine C) AST, ALT, and GGT D) Hemoglobin, hematocrit, and red blood cell count

Ans: C Feedback: The FDA has issued a black box warning stating that ketoconazole can cause hepatotoxicity. As a result, monitoring of liver function may be a priority over assessment of white cells, red cells, and renal function.

A woman is prescribed hormonal contraceptives. What aspect of this treatment places her at greatest risk for the development of blood clots? A) Hormonal contraceptives increase serum triglyceride and cholesterol. B) Hormonal contraceptives increase platelet levels. C) Hormonal contraceptives increase blood levels of clotting factors. D) Hormonal contraceptives cause injury to blood vessel lumens.

Ans: C Feedback: The administration of estrogen in hormonal contraceptives increases hepatic production of blood clotting factors. Hormonal contraceptives will not increase serum triglyceride and cholesterol or platelet levels. Hormonal contraceptives will cause injury to blood vessels.

A woman is being administered IV magnesium sulfate. What is a desired outcome related to the administration of magnesium sulfate? A) Increased contractions B) Respiratory rate above 18 C) Decreased blood pressure D) Increased uterine tone

Ans: C Feedback: The administration of magnesium sulfate can prevent seizure activity and reduce severe hypertension. Magnesium sulfate is administered for preterm labor to prevent contractions and reduce uterine tone, not to increase respiratory rate.

The nurse is preparing to administer a vaccine to a newborn. Before administering the vaccine, the nurse should A) warm the vaccine to well above room temperature. B) vigorously massage the chosen injection site. C) check the infant's temperature. D) divide the dose for administration to three injection sites.

Ans: C Feedback: The nurse should check the infant's temperature before administering any vaccine. Three injection sites are not normally required, and vigorous massage is not indicated. Warming the vaccine is not normally necessary.

A public health nurse is responsible for the administration of numerous immunizations. Which of the following guidelines regarding anaphylaxis should the nurse adhere to? A) The patient should be observed for anaphylaxis for 1 minute after administration. B) The patient should be observed for anaphylaxis for 5 minutes after administration. C) The patient should be observed for anaphylaxis for 30 minutes after administration. D) The patient should be observed for anaphylaxis for 90 minutes after administration.

Ans: C Feedback: The nurse should observe for allergic reactions, which usually occur within 30 minutes. The patient should be observed longer than 1 or 5 minutes, and it is not necessary to observe for longer than 30 minutes.

A 34-year-old woman has presented to the clinic for the first time, and the nurse learns that she has been taking Depo Provera for the past 13 years. This aspect of the woman's medical history should prompt what assessment? A) Cardiac stress testing B) Renal ultrasound C) Bone density testing D) Evaluation of triglyceride levels

Ans: C Feedback: There is a black box warning regarding the use of Depo Provera and Depo-SubQ Provera that relates to its being associated with the loss of stored calcium from bones, which increases the risk of broken bones and osteoporosis, especially after menopause. There is a greater risk of bone loss with long-term use. Consequently, bone density testing may be warranted. There is no particular indication for this patient to undergo stress testing, renal ultrasound, or triglyceride testing.

A patient has been diagnosed with hypothyroidism and admits to the nurse that she has heard of her thyroid gland but does not know the function of thyroid hormone. The nurse should explain the fact that thyroid hormone is responsible for A) regulating the levels of most other hormones in the body. B) stimulating the brain and sex organs. C) controlling the rate of cell metabolism throughout the body. D) regulating levels of glucose in the blood and body tissues.

Ans: C Feedback: Thyroid hormones control the rate of cellular metabolism and thus influence the functioning of virtually every cell in the body. The heart, skeletal muscle, liver, and kidneys are especially responsive to the stimulating effects of thyroid hormones. The brain, spleen, and gonads are less responsive. Thyroid hormone does not primarily influence glucose levels or the function of the endocrine system.

A woman who began labor several hours ago is to be administered oxytocin. What is the goal of oxytocin therapy? A) Prevent postpartum bleeding B) Decrease fetal hyperactivity C) Augment weak or irregular contractions D) Diminish periods of relaxation

Ans: C Feedback: Use of this manufactured hormone induces labor or augments weak, irregular uterine contractions during labor. It is not used in the labor phase to prevent bleeding. It is not administered to decrease fetal hyperactivity. The administration of oxytocin should allow for adequate periods of relaxation between contractions.

During a teaching session on the care of the diabetic patient, a family member asks why her daughter has a different insulin than her best friend. The nurse should make which of the following statements to explain the differences in insulin? A) "Insulin is prescribed based on the insurer's criteria for reimbursement." B) "Insulin is prescribed based on the patient's age." C) "Insulins have different onsets and durations of action." D) "Insulin type is matched with the appropriate oral hypoglycemic agent."

Ans: C Feedback: When insulin therapy is indicated, the physician may choose from several preparations that vary in composition, onset, duration of action, and other characteristics. Insulin is not prescribed based solely on cost. Insulin is not prescribed based solely on the patient's age. Insulin is not usually matched with oral hypoglycemic agents.

A child has been referred to a pediatric endocrinologist and has begun treatment with octreotide acetate. What nursing diagnosis should be noted in the child's plan of nursing care? A) Delayed growth and development B) Excess fluid volume C) Risk for deficient fluid volume D) Altered growth and development

Ans: D Feedback: Acromegaly is the major clinical indication for the use octreotide. In this condition, growth is excessive, not delayed. Fluid volume is not directly affected.

An adult patient has been experiencing severe lethargy and fatigue over the past several days, and she describes herself as feeling "shaky." Random blood glucose testing reveals a glucose level of 38 mg/dL, but the woman denies any significant change in her diet. The care team should suspect the possibility of what health problem? A) Chronic renal failure B) Acute renal failure C) Cushing's disease D) Adrenal insufficiency

Ans: D Feedback: Any patient with unexplained severe hypoglycemia requires assessment for adrenal insufficiency. Cushing's disease and kidney disease do not result in hypoglycemia.

When a patient has an increased serum level of ionized calcium, which hormone will be released? A) Insulin B) Estrogen C) PTH D) Calcitonin

Ans: D Feedback: Calcitonin is a hormone from the thyroid gland whose secretion is controlled by the concentration of ionized calcium in the blood flowing through the thyroid gland. When the serum level of ionized calcium is increased, secretion of calcitonin is increased. The serum level of calcium will not affect the release of insulin or estrogen. PTH is a response to low calcium levels.

A woman is at 42 weeks of gestation. Which of the following medications will be administered to promote cervical ripening? A) Calcium gluconate B) Magnesium sulfate C) Terbutaline (Brethine) D) Dinoprostone (Cervidil)

Ans: D Feedback: Cervidil is administered to ripen the cervix in a woman who is at 42 weeks of gestation. Calcium gluconate, magnesium sulfate, and terbutaline are not administered to ripen the cervix.

A 37-year-old woman has just experienced the end of a long-term relationship and has expressed a desire to begin using oral contraceptives. What aspect of this woman's current health status would contraindicate this treatment? A) The woman has a diagnosis of type 2 diabetes. B) The woman has asthma and uses inhaled corticosteroids and bronchodilators. C) The woman has a family history of breast cancer. D) The woman takes an antiplatelet medication for coronary artery disease.

Ans: D Feedback: Coronary artery disease is a contraindication to the use of oral contraceptives. Diabetes and asthma do not necessarily contraindicate this treatment. A family history of breast cancer is relevant but would not absolutely contraindicate oral contraceptive use.

A patient is suspected of having Cushing's disease. What hormone is used in the diagnosis of this disorder? A) Growth hormone B) Adrenocorticotropic hormone C) Thyroid-stimulating hormone D) Corticotropin-releasing hormone

Ans: D Feedback: Corticotropin-releasing hormone can be used in the diagnosis of Cushing's disease. Growth hormone is released in response to low blood levels of growth hormone. Corticotropin-releasing hormone causes the release of corticotropin (adrenocorticotropic hormone) in response to stress. Thyroid-stimulating hormone regulates the secretion of thyroid hormones.

A patient with thalassemia requires frequent blood transfusions. The patient develops iron overload. Which of the following medications will be administered? A) Calcium gluconate B) Deferoxamine (Desferal) C) Sodium polystyrene sulfonate (Kayexalate) D) Deferasirox (Exjade)

Ans: D Feedback: Deferasirox (Exjade) is an oral iron chelating agent that is used to treat chronic iron overload in patients who require frequent blood transfusions for severe chronic anemia. Calcium gluconate, deferoxamine (Desferal), and sodium polystyrene sulfonate (Kayexalate) are not used to treat chronic iron overload.

An adult patient is experiencing deficiencies in folic acid and vitamin B12. This patient's compromised health status creates a risk for which of the following? A) Hyperuricemia B) Hepatitis C) Non-Hodgkin's lymphoma D) Megaloblastic anemia

Ans: D Feedback: Deficiency states of both vitamin B12 and folic acid present similarly as megaloblastic anemia (characterized by abnormally large, immature red blood cells). These deficiencies do not contribute to hyperuricemia, hepatitis, or lymphoma.

An elderly patient has been diagnosed with Paget's disease. Which serum electrolyte is altered in this disease process? A) Sodium B) Potassium C) Chloride D) Calcium

Ans: D Feedback: Disorders of calcium and bone metabolism include hypocalcemia, hypercalcemia, osteoporosis, Paget's disease, and bone breakdown associated with breast cancer and multiple myeloma. Sodium, potassium, and chloride alterations are not primarily associated with Paget's disease.

A patient with a long-standing diagnosis of Addison's disease has been taking hydrocortisone for several years, achieving adequate symptom control. In recent weeks, the patient has experienced profound stress resulting from the collapse of his small business and subsequent conflict with his business partner. How might the presence of these stressors affect the patient's medication regimen? A) The patient should take his scheduled hydrocortisone later in the day. B) The ratio of mineralocorticoids to glucocorticoids should be adjusted. C) The patient's hydrocortisone should be temporarily withheld. D) The patient may temporarily require a higher dose of hydrocortisone.

Ans: D Feedback: During times of increased stress, the dosage of the medication may need to be increased. There would be no need to change the ratio of mineralocorticoids to glucocorticoids or to take the medications later in the day.

A patient is being administered magnesium sulfate for preterm labor. The patient's serum magnesium level is elevated at 11 mg/dL. With what sign or symptom will the patient likely present? A) Tachypnea B) Muscle rigidity C) Tachycardia D) Depressed deep tendon reflexes

Ans: D Feedback: Hypermagnesemia will cause depressed deep tendon reflexes. The serum magnesium level of 11 mg/dL would result in depressed respirations. The serum magnesium level of 11 would result in decreased muscle strength. The serum level of 11 mg/dL would result in bradycardia, not tachycardia.

During patient teaching, a young woman asks the nurse the following question: "If I get pregnant on the 'pill,' should I continue to take it?" What is the nurse's best response? A) "The pill has no effect on pregnancy." B) "The pill will cause miscarriage." C) "This is a personal choice for each woman." D) "The pill can be harmful to the fetus and should be discontinued."

Ans: D Feedback: If pregnancy does occur, estrogens are contraindicated because their use during pregnancy has been associated with the occurrence of vaginal cancer in female offspring. The pill will affect the pregnancy and should be discontinued. The pill will not cause miscarriage. The pill can cause harmful effects on male offspring if it is continued through pregnancy.

Sildenafil has been prescribed to a patient in his 60s, and the nurse is providing health education about the safe and effective use of the drug. The nurse should teach the patient to A) avoid taking sildenafil if he experiences heartburn. B) take the drug every 30 minutes if necessary, to a maximum of three doses. C) take the drug on an empty stomach. D) avoid taking the drug more than once in any 24-hour period.

Ans: D Feedback: It is important not to take sildenafil more than once in a 24-hour period. A recent high-fat meal may delay drug action, but it is not essential to take it on an empty stomach. Heartburn is not a common adverse effect, and would not necessarily require discontinuing treatment.

A girl who is 7 1/2 years old has received a diagnosis of precocious puberty from the pediatric endocrinologist. The nurse who collaborates with this physician should anticipate the administration of what drug? A) Menotropins (Pergonal) B) Thyrotropin alfa (Thyrogen) C) Cosyntropin (Cortrosyn D) Leuprolide acetate (Lupron)

Ans: D Feedback: Leuprolide is useful for the treatment of central precocious puberty in children. Menotropins (Pergonal) stimulates ovulation. Cosyntropin (Cortrosyn) stimulates the adrenal cortex to synthesize and secrete adrenocortical hormones. Thyrotropin alfa (Thyrogen) stimulates the secretion of thyroglobulin.

When providing patient teaching to parents regarding measles, mumps, and rubella vaccine administration, which of the following is most important regarding the schedule for administration? A) It is administered at 1 to 2 months. B) It is administered at 3 to 4 months. C) It is administered at 5 to 6 months. D) It is administered at 12 to 15 months.

Ans: D Feedback: Measles, mumps, and rubella immunization is administered initially at 12 to 15 months of age. The vaccine is not administered under the age of 1 year.

A patient is in diabetic ketoacidosis. The patient blood glucose level is over 600 mg/dL. The physician has ordered the patient to receive an initial dose of 25 units of insulin intravenously. What type of insulin will most likely be administered? A) NPH insulin B) Lente insulin C) Ultralente insulin D) Regular insulin

Ans: D Feedback: Regular insulin has rapid onset of action and can be given via IV. It is the drug of choice for acute situations, such as diabetic ketoacidosis. Isophane insulin (NPH) is used for long-term insulin therapy. Lente insulin is an intermediate-acting insulin. Ultralente insulin is a long-acting insulin.

A patient has received a rubella immunization. The patient was unaware that she was pregnant. What risk is associated with the administration of the rubella immunization in this patient? A) Risk of development of the disease in the newborn B) Risk of low infant birth weight C) Risk of preterm labor D) Risk of birth defects

Ans: D Feedback: Rubella during the first trimester of pregnancy is associated with a high incidence of birth defects in the newborn. Rubella is not associated with the development of disease. Rubella is not associated with low birth weight or preterm labor.

An infant is seen in the clinic for her first immunizations. When providing patient teaching to the parent, which of the following is no longer recommended for administration? A) Rubella and mumps vaccine B) Polio vaccine C) Diphtheria, pertussis, and tetanus vaccine D) Smallpox vaccine

Ans: D Feedback: Smallpox has been largely eradicated and is no longer administered to children. By 4 to 6 years of age, children should have received vaccinations for chickenpox, diphtheria, hepatitis A and B, influenza, measles, mumps, pertussis, polio, pneumococcal diseases, rubella, tetanus, and Haemophilus influenzae and rotavirus infections.

A patient has been diagnosed with hyperthyroidism. She is scheduled to receive a medication to destroy the thyroid gland. Which medication will be administered? A) Propylthiouracil (PTU) B) Methimazole (Tapazole) C) Saturated solution of potassium iodide (SSKI) D) Sodium iodide 131I (Iodotope)

Ans: D Feedback: Sodium iodide 131I (Iodotope) is a radioactive isotope of iodine. Therapeutic doses act by emitting beta and gamma rays, which destroy thyroid tissue and thereby decrease the production of thyroid hormones. Propylthiouracil (PTU) is not administered to destroy thyroid tissue. Methimazole (Tapazole) is not administered to destroy thyroid tissue. Saturated solution of potassium iodide (SSKI) is not administered to destroy thyroid tissue.

An adolescent is being treated with growth hormone. When assessing for potential adverse effects, what assessment should the nurse prioritize? A) Deep tendon reflexes B) Level of anxiety C) Respiratory rate D) Blood glucose

Ans: D Feedback: Somatropin decreases insulin sensitivity, resulting in hyperglycemia. Patients most at risk for increased insulin sensitivity are those with obesity, Turner's syndrome, or a family history of diabetes mellitus. Growth hormone can cause anxiety, but is not a priority assessment in this case. The respiratory rate is not the most important objective assessment in this case.

A patient is being treated for preterm labor. Which beta-adrenergic medication is administered orally to decrease uterine contractions? A) Magnesium sulfate B) Oxytocin (Pitocin) C) Nifedipine (Procardia) D) Terbutaline (Brethine)

Ans: D Feedback: Terbutaline is a beta-adrenergic agent that inhibits uterine contractions by reducing intracellular calcium levels; oral doses can be given as maintenance therapy. Magnesium sulfate is administered intravenously and not administered at home. Oxytocin induces labor. Nifedipine is a calcium channel blocker that decreases uterine contractions.

A woman in preterm labor has been administered terbutaline sulfate (Brethine). For what potential adverse effects should the nurse assess the patient? A) Pruritus (itching) and copious diaphoresis B) Joint pain and numbness in her extremities C) Headache and visual disturbances D) Palpitations and shortness of breath

Ans: D Feedback: Terbutaline sulfate (Brethine) is a beta-adrenergic agent that inhibits uterine contractions by reducing intracellular calcium levels. Adverse effects may include hyperkalemia, hyperglycemia, cardiac dysrhythmias, hypotension, and pulmonary edema. Women commonly experience hand tremors, palpitations, and shortness of breath with chest tightness.

A nurse is administering a mumps vaccine to an adolescent. Which of the following medications should be available when administering an immunization? A) Diphenhydramine (Benadryl) B) Hydroxyzine (Vistaril) C) Physostigmine D) Epinephrine

Ans: D Feedback: The administration of vaccines for immunization possesses the risk of an allergic reaction and anaphylaxis. The nurse should have aqueous epinephrine available in the event of an anaphylactic reaction. The administration of diphenhydramine or hydroxyzine will reduce the allergic reaction but will not be effective in the event of anaphylaxis. Physostigmine is not administered.

It is important for the nurse to stay informed of the most current recommendations for immunizations. Which of the following sources is most accurate regarding immunization guidelines? A) American Academy of Pediatrics B) American Academy of Family Physicians C) American Academy of Infectious Disease Physicians D) Centers for Disease Control and Prevention

Ans: D Feedback: The best source of information for current recommendations is the Centers for Disease Control and Prevention. The American Academy of Pediatrics, American Academy of Family Physicians, and American Academy of Infectious Disease Physicians are all sources of immunization information, but their information is based on the CDC's information.

When providing a health promotion presentation to a group of seniors, how often should you instruct the senior group to obtain influenza vaccines? A) One time only B) Every 10 years C) Two times per year D) Yearly

Ans: D Feedback: The influenza vaccine should be administered yearly. It is not administered as a single dose, every 10 years, or two times per year.

A patient suffers from numbness and tingling around the mouth and has a positive Trousseau's sign after a thyroidectomy. Which of the following medications will be administered? A) Parenteral normal saline B) Parenteral potassium chloride C) Parenteral digoxin (Lanoxin) D) Parenteral calcium gluconate

Ans: D Feedback: The patient has acute hypocalcemia as evidenced by numbness and tingling around the mouth and a positive Trousseau's sign. An intravenous calcium salt, such as calcium gluconate, is given for acute symptomatic hypocalcemia. Parenteral normal saline may be administered, but it will not increase the calcium level. The administration of potassium chloride parenterally must be diluted and will not affect the calcium level. Parenteral digoxin (Lanoxin) will not be administered.

A 75-year-old patient with overall good health is to begin taking vitamin D supplements. What is the recommended supplement for her age? A) 100 International Units daily B) 200 International Units daily C) 600 International Units daily D) 800 International Units daily

Ans: D Feedback: The recommended dietary allowance, or RDA, for vitamin D is 600 International Units for people 1 to 70 years of age and 800 International Units daily for adults 71 years and older to prevent and treat osteoporosis.

A patient with alcoholism has been admitted to the hospital following an injury. Assessment of the patient's overall health reveals significant nutritional deficits. The patient is likely to require which of the following due to inadequate dietary intake? A) Sodium B) Vitamin A C) Vitamin K D) Thiamine

Ans: D Feedback: Thiamine deficiency is common in patients with alcoholism because of inadequate dietary intake and use of large amounts of thiamine to metabolize ethanol. Sodium, vitamin A, and vitamin K are important but not as crucial to this particular patient as thiamine.

A nurse is teaching a young adult patient about oral contraceptives, which the patient will soon begin taking. The patient's primary concern is the effectiveness of the medication in preventing pregnancy. The nurse should teach the patient that oral contraceptives, when taken consistently, are how effective in preventing pregnancy? A) Between 96% and 98% B) Approximately 98.5% C) Highly effective, but partially dependent on genetic factors D) Nearly 100%

Ans: D Feedback: When taken correctly, estrogen-progestin contraceptive preparations are nearly 100% effective in preventing pregnancy. Genetics do not significantly affect treatment.

A patient is being administered propranolol (Inderal) to treat hypertension related to hyperthyroidism. Which condition will warrant the tapering and discontinuation of the propranolol (Inderal)? A) Hyperthyroidism B) Hypertension C) Angina pectoris D) Euthyroid state

Ans: D Feedback: When the patient becomes euthyroid and hyperthyroid symptoms are controlled by definitive treatment measures, propranolol should be tapered and discontinued. Hyperthyroidism, hypertension, and angina pectoris all warrant the administration of propranolol (Inderal).

A woman of childbearing age is diagnosed with breast cancer. She is currently taking hormonal contraceptives. What information should the woman be given regarding the hormonal contraceptives? A) The hormonal contraceptives do not affect the course of the disease. B) The hormonal contraceptives will stimulate uterine bleeding. C) The hormonal contraceptives will impair uterine bleeding. D) The hormonal contraceptives will stimulate tumor growth.

Ans: D Feedback: With known or suspected cancers of the breast or genital tissues, hormonal contraceptives may stimulate tumor growth. The hormonal contraceptives should be discontinued due to adverse reactions and stimulated tumor growth. The hormonal contraceptives will not inhibit tumor growth. The hormonal contraceptives can decrease uterine bleeding, but this action is not related to the breast tumor. The hormonal contraceptives will not stimulate or impair uterine bleeding in relation to the breast mass.

12. A 70-year-old man has enjoyed good overall health for all of his adult life, but he has been experiencing urinary frequency and dribbling that has culminated in a diagnosis of benign prostatic hypertrophy (BPH). As a result, the patient has been prescribed finasteride (Proscar). When teaching the patient about the potential adverse effects of the drug, the nurse should ensure that he knows about the possibility of A) sexual dysfunction. B) urethral burning. C) kidney stones. D) visual disturbances.

Ans: A Feedback: Adverse effects of finasteride include various sexual dysfunctions, such as impotence, gynecomastia, reduced libido, and ejaculatory disorders. Urethral burning, kidney stones, and visual disturbances are not common adverse effects of finasteride.

An older adult man has moved to a long-term care facility and the nurse is performing a medication reconciliation. The resident's current medication regimen includes alfuzosin (Uroxatral). After considering the most likely indication for this drug, the nurse should include what potential problem in the resident's interdisciplinary plan of care? A) Impaired urinary elimination B) Ineffective sexual pattern C) Sexual dysfunction D) Functional urinary incontinence

Ans: A Feedback: Alfuzosin (Uroxatral) is classed as a nonreceptor subtype selective alpha1-adrenergic blocker also used to treat BPH. Impaired urinary elimination is the hallmark of BPH. Incontinence may result from BPH, but it is less commonly the functional subtype.

An older adult patient has been using levothyroxine (Synthroid) for several years on an outpatient basis. The patient has recently sought care, with complaints that are consistent with hypothyroidism. To which of the patient's following statements may the nurse attribute the decreased effect of Synthroid? A) "I've been using a lot of antacids lately because of my indigestion." B) "My daughter and I have started the Atkin's diet to try to lose some weight." C) "Overall, I'd say that I'm under a lot of stress lately." D) "I've stopped taking aspirin for my arthritis and started using Tylenol."

Ans: A Feedback: Antacids may decrease the effect of levothyroxine. Acetaminophen, stress, and high protein intake do not have this effect.

A patient has been using Viagra on an intermittent basis for several years. However, he has cited delays in the onset and peak of action as the occasional source of frustration. What PDE5 inhibitor may be of particular benefit to this patient's needs? A) Avanafil (Stendra) B) Tadalafil (Cialis) C) Vardenafil (Levitra) D) Alprostadil (Caverject)

Ans: A Feedback: Avanafil (Stendra), the newest PDE5 inhibitor, is a fast-acting and highly selective member of the class with a peak in 30 minutes. Cialis and Levitra have longer onsets of action and peaks. Alprostadil (Caverject) is not a PDE5 inhibitor.

A patient will soon begin treatment for diabetes using glyburide. Which of the following conditions must be met in order for treatment to be effective? A) The patient must have functioning pancreatic beta cells. B) The patient must have hemoglobin A1C of 7%. C) The patient must not have hyperglycemia. D) The patient must be able to self-administer the medication.

Ans: A Feedback: Because glyburide stimulates pancreatic beta cells to produce more insulin, it is effective only when functioning pancreatic beta cells are present. The presence of normal blood glucose levels would render the medication unnecessary. Self-administration is common but not absolutely necessary.

A 41-year-old male patient with a complex medical history has been referred to the endocrinology department. Diagnostic testing and assessment have resulted in a diagnosis of secondary hypogonadism. Which of the following health problems is the most likely etiology of his diagnosis? A) Type 1 diabetes B) Mumps C) An inflammatory process in the testicles D) Testicular trauma

Ans: A Feedback: Chronic diseases (e.g., metabolic syndrome, diabetes) can lead to secondary hypogonadism. Common diseases that can cause primary hypogonadism are mumps, testicular inflammation, and trauma.

A patient's recent symptoms of unexplained fatigue and listlessness have prompted a referral to the endocrinology department. A plasma corticotropin (ACTH) stimulation test for adrenal insufficiency is being performed. Following the administration of ACTH, what finding would confirm the suspected diagnosis? A) Low cortisol levels B) Stupor C) Increased levels of CRH D) Signs of Cushing's disease

Ans: A Feedback: Confirming the diagnosis of adrenocortical insufficiency requires a short plasma corticotropin (ACTH) stimulation test. The examiner administers corticotropin in the morning, and a subnormal blood cortisol level in the morning and afternoon confirms the diagnosis. CRH is a precursor to ACTH, and would not be directly affected. Cushing's disease is adrenocortical excess, with high cortisol levels.

A patient is suspected of having adrenal insufficiency. Which hormone may be administered to diagnose Addison's disease? A) Corticotropin (ACTH) B) Somatotropin (Humatrope) C) Octreotide (Sandostatin) D) Leuprolide (Lupron)

Ans: A Feedback: Corticotropin (ACTH) is sometimes used as a diagnostic test to differentiate primary adrenal insufficiency (Addison's disease). Somatotropin is therapeutically equivalent to endogenous growth hormone. Octreotide (Sandostatin) is used for acromegaly. Leuprolide causes a decrease in testosterone and estrogen.

A 4-year-old child has eaten many of his mother's prenatal vitamins. Which medication is indicated for iron toxicity? A) Deferoxamine (Desferal) B) Penicillamine (Cuprimine) C) Sucralfate (Carafate) D) Magnesium sulfate

Ans: A Feedback: Deferoxamine is a parenteral drug used to remove excess iron from storage sites in the body. Penicillamine, sucralfate, and magnesium sulfate are not administered for iron toxicity.

A patient will soon begin long-term treatment with hydrocortisone after being diagnosed with Addison's disease. In order to mitigate the potentially adverse effect of this treatment regimen, the nurse should encourage the patient to increase her intake of what nutrient? A) Water B) Calcium C) Sodium D) Potassium

Ans: B Feedback: Due to the potential for reduced bone density, patients should be encouraged to increase calcium intake if hydrocortisone is administered for a prolonged period. Increased sodium, potassium, and water intake are not recommended.

A pregnant woman asks why she needs to take a folic acid supplement. What is the nurse's best explanation for the administration of folic acid? A) "Folic acid prevents the development of contractions." B) "Folic acid prevents neural tube birth defects." C) "Folic acid builds strong fetal bones." D) "Folic acid will decrease nausea and vomiting."

Ans: B Feedback: Folic acid prevents neural tube birth defects. Folic acid does not prevent contractions. Folic acid will not build fetal bones. Folic acid will not prevent nausea and vomiting.

A child is born with cretinism. What element was lacking in the mother's diet during pregnancy? A) Potassium B) Iodine C) Sodium D) Magnesium

Ans: B Feedback: Cretinism is uncommon in the United States but may occur with a lack of iodine in the mother's diet. A lack of potassium, sodium, and magnesium in the mother's diet will not cause cretinism.

A patient with a diagnosis of Cushing's disease has been admitted to the emergency department after taking a fall on the sidewalk outside her apartment building. This patient's underlying disease process creates increased risks of A) hemorrhage and impaired hemostasis. B) fractures and impaired wound healing. C) neurovascular complications and rhabdomyolysis. D) bruising and hematoma.

Ans: B Feedback: Cushing's disease results in low bone density and impaired wound healing. It does not typically cause impaired hemostasis or neurovascular problems.

An adult patient is suspected of having an androgen deficiency and has spoken with his primary care provider about the possibility of treatment with testosterone. The use of testosterone would be most complicated by the presence of what preexisting health problem? A) Urinary incontinence B) BPH C) Chronic renal failure D) Type 2 diabetes

Ans: B Feedback: Disorders of the prostate contraindicate testosterone use because men with an enlarged prostate may have additional enlargement. Incontinence, diabetes, and kidney disease are not absolute contraindications to the use of testosterone.

A pregnant woman states that she has been constipated since becoming pregnant. Which medication is most appropriate for preventing constipation related to pregnancy? A) Metamucil B) Mineral oil C) Saline cathartic D) Stimulant cathartic

Ans: A Feedback: A bulk-producing agent, such as Metamucil, is most physiologic for the mother and safe for the fetus. Mineral oil is not recommended because of the lack of absorption of fat-soluble vitamins. Saline cathartics are not recommended because of hypernatremia. Stimulant cathartics are not recommended for the pregnant woman.

A patient has been diagnosed with myxedema. She currently takes ibuprofen (Motrin) for knee pain. For what is this patient at increased risk? A) Nephrotoxicity B) Pulmonary congestion C) Tachycardia D) Apnea

Ans: A Feedback: A patient diagnosed with myxedema cannot metabolize and excrete drugs, so she is at an increased risk of developing nephrotoxicity. Pulmonary congestion, tachycardia, and apnea are not adverse effects related to myxedema or the administration of ibuprofen (Motrin).

A new mother asks why her baby receives vitamin K IM at birth. What is the best statement to explain the need for vitamin K in a newborn? A) "The administration of vitamin K prevents her from bleeding." B) "Your infant will need multiple injections of vitamin K to prevent health problems." C) "Your infant will need the conjugated iron in vitamin K to protect her from infection." D) "Vitamin K helps boost her nutrition until she's able to feed more efficiently."

Ans: A Feedback: A single IM dose of vitamin K is given to newborn infants to prevent hemorrhagic disease of newborns. The infant does not need multiple injections of vitamin K to prevent hemorrhage. The administration of vitamin K will not protect from infection. Vitamin K is not used to enhance nutrition.

A 38-year-old patient has been living with Addison's disease for many years and has achieved adequate symptom control through lifestyle modifications and drug therapy. However, the patient has now been admitted to the hospital in addisonian crisis. Which of the nurse's following assessment questions is most appropriate? A) "Have you been under an unusual amount of stress lately?" B) "How would you describe your diet over the last few days?" C) "When was the last time that you drank any alcohol?" D) "Has your urine output been typical for the past few days?"

Ans: A Feedback: Acute adrenal crisis, or addisonian crisis, is a life-threatening condition that occurs when Addison's disease is the underlying problem and the patient is exposed to minor illness or increased stress. Diet, alcohol use, and impaired urinary function are less likely contributors.

A 17-year-old boy has been admitted to the emergency department after suffering a knee sprain during a football practice. The nurse notes that the patient has exceptional muscle mass, and the boy quietly admits that this is mostly attributable to the use of steroids, which he obtains from a teammate. What should the nurse teach the boy about anabolic steroid abuse? A) The ability of anabolic steroids to build muscle is greatly exaggerated in the media. B) Anabolic steroids are universally dangerous but are especially harmful to adolescents. C) The muscle mass resulting from steroid use will atrophy unless doses are continually increased. D) Anabolic steroids will reduce the boy's ability to perform weight-bearing exercise later in life.

Ans: B Feedback: Although steroids have a reputation for being dangerous to adult athletes, such as body builders a`nd football players, they are considered even more dangerous for teens because teens are still growing. These drugs do not necessarily require increasing doses to be effective. They carry numerous health consequences later in life, but reduced weight bearing is not among the most common consequences.

A patient in his mid-30s has received a diagnosis of type 2 diabetes. Following his diagnosis, he has been meeting with a nurse regularly as well as performing extensive online research. Which of the patient's statements should prompt the nurse to perform further teaching? A) "I don't like getting this diagnosis, but I know that treatment now can prevent future health consequences." B) "I'm disappointed, but I take some solace in the fact that I won't ever have to have insulin injections." C) "People always tried to encourage me to lose weight, and I suppose they might have been right." D) "From what I've learned, I know that the basic problem is that my pancreas can't keep up with my insulin needs."

Ans: B Feedback: Among people with type 2 diabetes, 20% to 30% require exogenous insulin at some point in their lives. Obesity is a major cause, and vigilant treatment can prevent future sequelae. The essence of type 2 diabetes is the pancreas' inability to meet insulin needs.

A patient has been involved in a motor vehicle accident. Which hormone will be released in response to his blood loss? A) Luteinizing hormone (LH) B) Antidiuretic hormone (ADH) C) Oxytocin D) Melanocyte-stimulating hormone

Ans: B Feedback: Antidiuretic hormone is also called vasopressin and is released in response to blood loss. Luteinizing hormone is important in ovulation. Oxytocin functions in childbirth and lactation. Melanocyte-stimulating hormone plays a role in skin pigmentation.

A female patient is taking combined hormonal contraceptives to prevent pregnancy. She visits the gynecology clinic and is noted to have a blood pressure of 176/102 mm Hg. The patient is started on enalapril mesylate 10 mg. In collaboration with the primary care provider, what other patient teaching should be provided based on her current medication regimen? A) Instruct on a no-salt diet. B) Instruct to discontinue the contraceptives. C) Instruct on the use of relaxation to decrease stress. D) Instruct on the rationale for increasing contraceptive dose.

Ans: B Feedback: Because of widespread effects on the body tissues and reported adverse reactions, the patient should be instructed to discontinue hormonal contraceptives that contribute to hypertension. The patient should restrict salt but not eliminate salt. The implementation of relaxation techniques is important but not crucial to the current medical regimen.

A patient's low ionized calcium levels have necessitated an infusion of an IV calcium preparation. During the administration of this medication, the nurse should prioritize what assessment? A) Oxygen saturation levels B) Cardiac monitoring C) Assessment of cognition D) Assessment for visual changes

Ans: B Feedback: Calcium imbalances can result in alterations in heart rate and rhythm; cardiac monitoring is thus necessary. The patient is less likely to experience alterations in respiratory function, cognition, or vision.

A 52-year-old has made an appointment with his primary care provider and has reluctantly admitted that his primary health concern is erectile dysfunction (ED). He describes the problem as increasing in severity and consequent distress. Which of the nurse's assessment questions is most likely to address a common cause of ED? A) "How would you describe your overall level of health?" B) "Are you taking any medications for high blood pressure?" C) "How has this problem been affecting your relationship with your wife?" D) "Have you suffered any injuries of any kind in the last several months?"

Ans: B Feedback: Causes of ED may include drugs such as antidepressants, antihypertensive agents, and histamine receptor antagonists. Injuries are a rare cause. Addressing the impact of the problem and the patient's perceptions of health are important, but neither question addresses causation.

A middle-aged patient with erectile dysfunction has sought care and is receiving health education from the nurses. The patient has expressed a desire to understand the physiology of the sex hormone production and the sexual response. The nurse should describe the fact that testosterone is normally secreted in response to A) sexual arousal. B) stimulation by luteinizing hormone. C) ACTH release by the adrenal cortex. D) decreased cortisol levels.

Ans: B Feedback: Certain cells in the testes, called Leydig cells, secrete testosterone in response to stimulation by luteinizing hormone (LH) from the anterior pituitary gland. Testosterone secretion is not the result of sexual arousal, ACTH, or low cortisol.

An adult patient has been diagnosed with a posterior pituitary lesion that has resulted in diabetes insipidus. The characteristic sign of this health problem is A) blood glucose levels 400 mg/dL. B) copious urine production. C) hyperglycemia that is unresponsive to exogenous insulin. D) hematuria.

Ans: B Feedback: Characteristic features of DM include polyuria, with excretion of dilute urine ranging from 4 L to as much as 30 L. Hematuria does not normally occur, and blood glucose levels are not directly affected.

A patient is scheduled for an exploratory surgery for cancer. What will cause the release of adrenocorticotropic hormone in response to this patient's psychological and physiological stress? A) Growth hormone release-inhibiting hormone (Somatostatin) B) Corticotropin-releasing hormone or factor (CRH or CRF) C) Growth hormone-releasing hormone (GHRH) D) Thyrotropin-releasing hormone (TRH)

Ans: B Feedback: Corticotropin-releasing hormone or factor (CRH or CRF) causes release of corticotropin (adrenocorticotropic hormone) in response to stress and threatening stimuli. Growth hormone release-inhibiting hormone (somatostatin) inhibits release of growth hormone. Growth hormone-releasing hormone causes the release of growth hormone in response to low blood levels of GH. Thyrotropin-releasing hormone causes the release of TSH in response to stress, such as cold.

A nurse educator is explaining the pathophysiology of diabetes to a newly diagnosed patient. The patient does not understand why she had a "constant, insatiable thirst" in the months preceding her diagnosis. What phenomenon should the nurse describe? A) "The excess glucose in your blood accumulates in your blood vessels and neurons, including the neurons that control thirst." B) "Excess glucose pulled more water through your kidneys and the increased urination caused thirst." C) "Increased thirst is your body's attempt to dilute your blood because it contains too much glucose." D) "When your body cells are starved for useful glucose, they signal your body to increase food and fluid intake."

Ans: B Feedback: When large amounts of glucose are present, water is pulled into the renal tubule. This results in a greatly increased urine output (polyuria). The excessive loss of fluid in urine leads to increased thirst (polydipsia). Glucose does not directly affect the thirst center.

A patient taking hormonal contraceptives will soon turn 35 years of age. She is moderately obese and has smoked for 15 years. Which of the following is most important? A) Begin an exercise regime. B) Begin smoking cessation. C) Begin daily aspirin therapy. D) Begin taking a loop diuretic to aid weight loss.

Ans: B Feedback: Women older than 35 years of age who smoke cigarettes are at greater risk for thromboembolic disorders. The patient should be instructed on smoking cessation. The patient should not begin an exercise regime unless prescribed by her physician. The patient should not begin daily aspirin or diuretic therapy unless prescribed by her physician.

When teaching new parents about the benefits of adhering to the recommended vaccination schedule, the nurse should cite protection against which of the following diseases? Select all that apply. A) HIV B) Measles C) Varicella D) Poliomyelitis E) Hepatitis B

Ans: B, C, D, E Feedback: Measles, varicella, polio, and HBV are all within the schedule of infant vaccinations. There is currently no vaccine against HIV.

A community health nurse is leading a health promotion workshop during a community health fair. A participant has asked the nurse for advice on the necessity of calcium supplements. The nurse should respond in the knowledge that which of the following demographic groups frequently have low calcium levels? Select all that apply. A) Elementary-aged children B) Young women C) Middle-aged men D) Middle-aged women E) Older women

Ans: B, E Feedback: Experts believe that the diets of most people of all ages, but especially of young women and older adults, are deficient in calcium.

When assessing for therapeutic effects of mitotane in a patient with adrenocortical carcinoma, the nurse should expect to identify A) a decrease in agitation. B) an audible S3. C) a decrease in blood pressure. D) a decrease in urine output.

Ans: C Feedback: A decrease in blood pressure is an expected therapeutic effect of mitotane. An audible S3 is not expected. Mitotane does not address agitation or high urine output.

A woman who is 7 months pregnant is waking up at night with gastroesophageal reflux. Which of the following medications is most highly recommended? A) Terbutaline (Brethine) B) Diphenoxylate (Lomotil) C) Ranitidine (Zantac) D) Chlorothiazide (Diuril)

Ans: C Feedback: A histamine2 receptor antagonist, such as ranitidine, is used for gastroesophageal reflux disease. Terbutaline is a tocolytic agent. Diphenoxylate is not administered for GERD. Chlorothiazide is a diuretic agent.

An insulin-dependent diabetic patient has begun taking an oral contraceptive. What effect will this medication regime have on her physiologically? A) Increase risk of hypoglycemia B) Increase heart rate C) Increase blood glucose D) Increase risk of metabolic alkalosis

Ans: C Feedback: A patient who has begun oral contraceptives and is insulin dependent will require more insulin due to increased blood glucose. The combination of oral contraceptives and insulin will not increase blood pressure or heart rate or cause metabolic alkalosis.

A patient has been prescribed acarbose (Precose). What is the advantage of acarbose over alternative drugs? A) It can replace the use of insulin. B) The patient does not have to limit food intake. C) It delays the digestion of complex carbohydrates. D) It prevents alkalosis.

Ans: C Feedback: Acarbose delays the digestion of complex carbohydrates into glucose and other simple sugars. Acarbose may be combined with insulin or an oral agent, usually a sulfonylurea. The patient will still need to remain on a diabetic dietary regime. The drug does not directly prevent acid-base imbalances.

Hydrocortisone has been prescribed to a patient for the treatment of Addison's disease, and the patient will soon be discharged. The nurse should encourage the patient to take this medication A) at least 30 minutes before or 2 hours after meals. B) at bedtime. C) before 9 AM D) with an antacid.

Ans: C Feedback: Administration of hydrocortisone should take place every morning before 9 AM. This minimizes HPA suppression. People should take the oral preparation with food to decrease gastric irritation.

When providing nutritional teaching to adolescent girls, which aspect of teaching is most important? A) Limit the amount of foods high in fiber. B) Increase potassium chloride in the diet. C) Calcium intake should be 1300 mg daily. D) Increase sodium in the diet.

Ans: C Feedback: Adolescent females should consume 1000 to 1300 mg of calcium per day. Adolescent females should not limit fiber or increase potassium chloride or sodium in the diet.

A patient has been prescribed alendronate (Fosamax). Which of the following instructions should the patient be given regarding the administration of this medication? A) Take the medication with whole milk. B) Take the medication and lie down for 30 minutes. C) Take the medication on an empty stomach. D) Take the medication to increase bone resorption.

Ans: C Feedback: Alendronate (Fosamax) must be taken on an empty stomach. Alendronate (Fosamax) should not be taken with milk or food. The patient should be sitting up or ambulatory for 30 minutes after taking the medication. The medication decreases bone resorption, not increases it.

A 72-year-old woman has been diagnosed with osteoporosis and has begun taking alendronate (Fosamax). The nurse should be aware that this drug can increase the woman's bone density by which of the following means? A) Decreasing renal excretion of calcium B) Increasing the binding of vitamin D to calcium ions C) Suppressing the function of osteoclasts D) Enhancing the function of osteoblasts

Ans: C Feedback: Alendronate suppresses osteoclast activity on newly formed resorption surfaces, which reduces bone turnover. This means that bone formation exceeds resorption at remodeling sites, leading to progressive gains in bone mass. Fosamax does not enhance osteoblast function or affect the renal excretion of calcium. It does not influence the chemical interaction between vitamin D and calcium.


Related study sets

Topic 2: Title VII of the Civil Rights Act of 1964

View Set

Ethical, Legal, and Professional Issues in Counseling - Part 2

View Set

Chapter 3: External Analysis: Industry Structure, Competitive Forces, and Strategic Groups

View Set

H&P ch. 6 Congenital diseases and disorders

View Set

Testout Network Pro Ch 1-8 Exams and Labs

View Set